PASS MED Flashcards

1
Q

A 27-year-old female presents with weakness of both hands. On examination you note wasting and weakness of the small muscles of the hands and loss of pain and temperature sensation over the trunk and arms. Vibration sense is intact.

What is the most likely diagnosis?

A

Syringomyelia - yringomyelia is a condition whereby fluid filled cavities develop within the spinal cord. Pressure can increase resulting in compression of the spinal cord tracts. The syrinx can extend to and damage the anterior horn cells, thereby resulting in lower motor neurone features. The spinothalamic tract axons decussate to the other side of the spinal cord via the anterior white commissure, and they are particularly susceptible to damage from the syrinx. Pain and temperature sensation are lost due to spinothalamic tract damage, and one side may be affected more than the other. Classically, the sensation loss is experienced in a shawl-like distribution over the arms, shoulders and upper body. Light touch, vibration and proprioception may also be affected as the syrinx enlarges into the dorsal columns.

How well did you know this?
1
Not at all
2
3
4
5
Perfectly
2
Q

Complications of aneurysmal SAH:

A

Re-bleeding
happens in around 10% of cases and most common in the first 12 hours
if rebleeding is suspected (e.g. sudden worsening of neurological symptoms) then a repeat CT should be arranged
associated with a high mortality (up to 70%)
Vasospasm (also termed delayed cerebral ischaemia), typically 7-14 days after onset
Hyponatraemia (most typically due to syndrome inappropriate anti-diuretic hormone (SIADH))
Seizures
Hydrocephalus
Death

How well did you know this?
1
Not at all
2
3
4
5
Perfectly
3
Q

window for thrombolysis stroke

A

A combination of thrombolysis AND thrombectomy is recommend for patients with an acute ischaemic stroke who present within 4.5 hours

NICE have also made recommendations to use a modified Rankin score of less than 3 and a National Institutes of Health Stroke Scale (NIHSS) score of more than 5 when considering the selection of patients for mechanical thrombectomy.

How well did you know this?
1
Not at all
2
3
4
5
Perfectly
4
Q

Contraindications to thrombolysis:

A

Absolute

  • Previous intracranial haemorrhage
  • Seizure at onset of stroke
  • Intracranial neoplasm
  • Suspected subarachnoid haemorrhage
  • Stroke or traumatic brain injury in preceding 3 months
  • Lumbar puncture in preceding 7 days
  • Gastrointestinal haemorrhage in preceding 3 weeks
  • Active bleeding
  • Pregnancy
  • Oesophageal varices
  • Uncontrolled hypertension >200/120mmHg

relative
Concurrent anticoagulation (INR >1.7)
- Haemorrhagic diathesis
- Active diabetic haemorrhagic retinopathy
- Suspected intracardiac thrombus
- Major surgery / trauma in the preceding 2 weeks

How well did you know this?
1
Not at all
2
3
4
5
Perfectly
5
Q

A 68-year-old gentleman was diagnosed with glaucoma and commenced on treatment. He returned in one month for review, his eye pressures had improved but he complains his eyelashes have increased in length. What medicine is likely to have caused his eyelashes to grow in length?

A

Lantanprost - treatment for glaucoma

How well did you know this?
1
Not at all
2
3
4
5
Perfectly
6
Q

preferred antiplatelet for secondary prevention following stroke

A

Clopidogrel is the preferred antiplatelet for secondary prevention following stroke

How well did you know this?
1
Not at all
2
3
4
5
Perfectly
7
Q
In baby (neonate) feeding intolerance, abdominal distension and bloody stools, which can quickly progress to abdominal discolouration, perforation and peritonitis.  
Xray - pneumatosis intestinalis (intramural gas)
A

Necrotising enterocolitis

How well did you know this?
1
Not at all
2
3
4
5
Perfectly
8
Q

Bone protection for patients who are going to take long-term steroids

A

should start immediately= start vitamin D and calcium supplements

How well did you know this?
1
Not at all
2
3
4
5
Perfectly
9
Q

tetanus vaccination - unsure history - management?

A

Patients with an uncertain tetanus vaccination history should be given a booster vaccine + immunoglobulin, unless the wound is very minor and < 6 hours old

How well did you know this?
1
Not at all
2
3
4
5
Perfectly
10
Q

Infective exacerbation of COPD antibiotics

A

Infective exacerbation of COPD: first-line antibiotics are amoxicillin or clarithromycin or doxycycline

How well did you know this?
1
Not at all
2
3
4
5
Perfectly
11
Q

Laryngomalacia

A

Congenital abnormality of the larynx.

Infants typical present at 4 weeks of age with:
stridor

How well did you know this?
1
Not at all
2
3
4
5
Perfectly
12
Q

Bacterial endocarditis - causative organism

A

he vast majority of cases of bacterial endocarditis are caused by gram positive cocci.

Common causes:
Streptococcus viridans
Staphylococcus aureus (in intravenous drugs uses or prosthetic valves)
Staphylococcus epidermidis (in prosthetic valves)

How well did you know this?
1
Not at all
2
3
4
5
Perfectly
13
Q

HBA1c diagnostic of diabetes.

A

A HbA1c of 48mmol/mol is diagnostic of diabetes.

Over 58 = 2nd medication

How well did you know this?
1
Not at all
2
3
4
5
Perfectly
14
Q

A 17-year-old male presents to the Ear Nose and Throat clinic to discuss the results from the fine needle aspirate taken from a smooth, fluctuant, non-tender, non-translucent mass located anterior to his left sternocleidomastoid muscle. It does not move on tongue protrusion. The aspirate reveals an acellular fluid with cholesterol crystals.

A

A branchial cyst is typically a benign, lateral, unilateral neck mass

How well did you know this?
1
Not at all
2
3
4
5
Perfectly
15
Q

VTE post warfarin length

A

Venous thromoboembolism - length of warfarin treatment
provoked (e.g. recent surgery): 3 months
unprovoked: 6 months

How well did you know this?
1
Not at all
2
3
4
5
Perfectly
16
Q

Pyloric stenosis

A

Pyloric stenosis typically presents in the second to fourth weeks of life with vomiting, although rarely may present later at up to four months. It is caused by hypertrophy of the circular muscles of the pylorus.

Epidemiology
incidence of 4 per 1,000 live births
4 times more common in males
10-15% of infants have a positive family history
first-borns are more commonly affected

Features
‘projectile’ vomiting, typically 30 minutes after a feed
constipation and dehydration may also be present
a palpable mass may be present in the upper abdomen
hypochloraemic, hypokalaemic alkalosis due to persistent vomiting

Diagnosis is most commonly made by ultrasound.

Management is with Ramstedt pyloromyotomy.

How well did you know this?
1
Not at all
2
3
4
5
Perfectly
17
Q

Myocardial infarction: secondary prevention

A

NICE produced guidelines on the management of patients following a myocardial infarction (MI) in 2013. Some key points are listed below

All patients should be offered the following drugs:
dual antiplatelet therapy (aspirin plus a second antiplatelet agent)
ACE inhibitor
beta-blocker
statin

How well did you know this?
1
Not at all
2
3
4
5
Perfectly
18
Q

Parotid gland mass - Benign pleomorphic adenoma or benign mixed tumor

A

Most common parotid neoplasm (80%)
Proliferation of epithelial and myoepithelial cells of the ducts and an increase in stromal components
Slow growing, lobular, and not well encapsulated
Recurrence rate of 1-5% with appropriate excision (parotidectomy)
Recurrence possibly secondary to capsular disruption during surgery
Malignant degeneration occurring in 2-10% of adenomas observed for long periods, with carcinoma ex-pleomorphic adenoma occurring most frequently as adenocarcinoma

How well did you know this?
1
Not at all
2
3
4
5
Perfectly
19
Q

Paranoid gland mass

A

Second most common benign parotid tumor (5%)
Most common bilateral benign neoplasm of the parotid
Marked male as compared to female predominance
Occurs later in life (sixth and seventh decades)
Presents as a lymphocytic infiltrate and cystic epithelial proliferation
May represent heterotopic salivary gland epithelial tissue trapped within intraparotid lymph nodes
Incidence of bilaterality and multicentricity of 10%
Malignant transformation rare (almost unheard of)

How well did you know this?
1
Not at all
2
3
4
5
Perfectly
20
Q

COPD management

A

General management
>smoking cessation advice: including offering nicotine replacement therapy, varenicline or bupropion
annual influenza vaccination
one-off pneumococcal vaccination
pulmonary rehabilitation to all people who view themselves as functionally disabled by COPD (usually Medical Research Council [MRC] grade 3 and above)

How well did you know this?
1
Not at all
2
3
4
5
Perfectly
21
Q

synchronised DC cardioversion indications

A

Unstable atrial fibrillation, atrial flutter, atrial tachycardia, and supraventricular tachycardias

How well did you know this?
1
Not at all
2
3
4
5
Perfectly
22
Q

Klebsiella pneumonia

A
Features of Klebsiella pneumonia
more common in alcoholic and diabetics
may occur following aspiration
'red-currant jelly' sputum
often affects upper lobes
How well did you know this?
1
Not at all
2
3
4
5
Perfectly
23
Q

Progressive multifocal leukoencephalopathy

A

Caused by JC or BK virus

How well did you know this?
1
Not at all
2
3
4
5
Perfectly
24
Q

superior vena cava obstruction presentation?

A
Features
dyspnoea is the most common symptom
swelling of the face, neck and arms - conjunctival and periorbital oedema may be seen
headache: often worse in the mornings
visual disturbance
pulseless jugular venous distension
How well did you know this?
1
Not at all
2
3
4
5
Perfectly
25
Q

superior vena Cava obstruction causes and management

A

Causes
common malignancies: small cell lung cancer, lymphoma
other malignancies: metastatic seminoma, Kaposi’s sarcoma, breast cancer
aortic aneurysm
mediastinal fibrosis
goitre
SVC thrombosis

Management
general: dexamethasone, balloon venoplasty, stenting
small cell: chemotherapy + radiotherapy
non-small cell: radiotherapy

How well did you know this?
1
Not at all
2
3
4
5
Perfectly
26
Q

ECG: hypokalaemia

A
ECG features of hypokalaemia
U waves
small or absent T waves (occasionally inversion)
prolong PR interval
ST depression
long QT
How well did you know this?
1
Not at all
2
3
4
5
Perfectly
27
Q

Palliative care prescribing: pain

A

when starting treatment, offer patients with advanced and progressive disease regular oral modified-release (MR) or oral immediate-release morphine (depending on patient preference), with oral immediate-release morphine for breakthrough pain

oral modified-release morphine should be used in preference to transdermal patches
laxatives should be prescribed for all patients initiating strong opioids
patients should be advised that nausea is often transient. If it persists then an antiemetic should be offered
drowsiness is usually transient - if it does not settle then adjustment of the dose should be considered

How well did you know this?
1
Not at all
2
3
4
5
Perfectly
28
Q

Palliative care prescribing: pain SIGN

A

the breakthrough dose of morphine is one-sixth the daily dose of morphine
all patients who receive opioids should be prescribed a laxative
opioids should be used with caution in patients with chronic kidney disease
oxycodone is preferred to morphine in palliative patients with mild-moderate renal impairment
if renal impairment is more severe, alfentanil, buprenorphine and fentanyl are preferred

How well did you know this?
1
Not at all
2
3
4
5
Perfectly
29
Q

Palliative care prescribing: pain - increasing dose

A

When increasing the dose of opioids the next dose should be increased by 30-50%.

How well did you know this?
1
Not at all
2
3
4
5
Perfectly
30
Q
Conversion between opioids
Codine -> morphine =
tramadol -> morphine 
Morphine -> oxy 
Oral morphine -> Subcutaneous morphine 
Oral morphine -> Subcutaneous dimorphine 
Oral oxycodone ->Subcutaneous dimorphine
A

Codine -> morphine = Divide by 10
tramadol -> morphne = Divide by 10

Morphine -> oxy = Divide by 1.5-2

Oral morphine -> Subcutaneous morphine = divide by 2
Oral morphine -> Subcutaneous dimorphine = divide by 3

Oral oxycodone ->Subcutaneous dimorphine = divide by 1.5

How well did you know this?
1
Not at all
2
3
4
5
Perfectly
31
Q

Small bowel bacterial overgrowth syndrome
Diagnosis
Management

A

Diagnosis
hydrogen breath test
small bowel aspiration and culture: this is used less often as invasive and results are often difficult to reproduce
clinicians may sometimes give a course of antibiotics as a diagnostic trial

Management
correction of underlying disorder
antibiotic therapy: rifaximin is now the treatment of choice due to relatively low resistance. Co-amoxiclav or metronidazole are also effective in the majority of patients.

How well did you know this?
1
Not at all
2
3
4
5
Perfectly
32
Q

Undescended testes

A

A congenital undescended testis is one that has failed to reach the bottom of the scrotum by 3 months of age
consider age - may watch and wait

Treatment
Orchidopexy at 6- 18 months of age. The operation usually consists of inguinal exploration, mobilisation of the testis and implantation into a dartos pouch.
Intra-abdominal testis should be evaluated laparoscopically and mobilised. Whether this is a single stage or two stage procedure depends upon the exact location.
After the age of 2 years in untreated individuals the Sertoli cells will degrade and those presenting late in teenage years may be better served by orchidectomy than to try and salvage a non functioning testis with an increased risk of malignancy.

How well did you know this?
1
Not at all
2
3
4
5
Perfectly
33
Q

epigastric ulcers

A

Features
epigastric pain
nausea
duodenal ulcers -more common than gastric ulcers
epigastric pain when hungry, relieved by eating

gastric ulcers- epigastric pain worsened by eating

How well did you know this?
1
Not at all
2
3
4
5
Perfectly
34
Q

non-invasive ventilation indications

A

Non-invasive ventilation - key indications
COPD with respiratory acidosis pH 7.25-7.35*
type II respiratory failure secondary to chest wall deformity, neuromuscular disease or obstructive sleep apnoea
cardiogenic pulmonary oedema unresponsive to CPAP
weaning from tracheal intubation

How well did you know this?
1
Not at all
2
3
4
5
Perfectly
35
Q

Keratoacanthoma

A

Features - said to look like a volcano or crater
initially a smooth dome-shaped papule
rapidly grows to become a crater centrally-filled with keratin

Spontaneous regression of keratoacanthoma within 3 months is common, often resulting in a scar. Such lesions should however be urgently excised as it is difficult clinically to exclude squamous cell carcinoma. Removal also may prevent scarring.

How well did you know this?
1
Not at all
2
3
4
5
Perfectly
36
Q

Osteoporosis: assessing risk

A

They advise that all women aged >= 65 years and all men aged >= 75 years should be assessed. Younger patients should be assessed in the presence of risk factors, such as:
previous fragility fracture
current use or frequent recent use of oral or systemic glucocorticoid
history of falls
family history of hip fracture
other causes of secondary osteoporosis
low body mass index (BMI) (less than 18.5 kg/m²)
smoking
alcohol intake of more than 14 units per week for women and more than 14 units per week for men.

Use FRAX

How well did you know this?
1
Not at all
2
3
4
5
Perfectly
37
Q

neuroleptic malignant syndrome management

A

Management
stop antipsychotic
patients should be transferred to a medical ward if they are on a psychiatric ward and often they are nursed in intensive care units
IV fluids to prevent renal failure
dantrolene may be useful in selected cases
thought to work by decreasing excitation-contraction coupling in skeletal muscle by binding to the ryanodine receptor, and decreasing the release of calcium from the sarcoplasmic reticulum
bromocriptine, dopamine agonist, may also be used

How well did you know this?
1
Not at all
2
3
4
5
Perfectly
38
Q

Gastro-oesophageal reflux disease presentation

A

oesophagitis secondary to refluxed gastric contents

How well did you know this?
1
Not at all
2
3
4
5
Perfectly
39
Q

Gastro-oesophageal reflux disease: management

A

Endoscopically proven oesophagitis
full dose proton pump inhibitor (PPI) for 1-2 months
if response then low dose treatment as required
if no response then double-dose PPI for 1 month

Endoscopically negative reflux disease
full dose PPI for 1 month
if response then offer low dose treatment, possibly on an as-required basis, with a limited number of repeat prescriptions
if no response then H2RA or prokinetic for one month

How well did you know this?
1
Not at all
2
3
4
5
Perfectly
40
Q

otitis externa Features and management

A

Features
ear pain, itch, discharge
otoscopy: red, swollen, or eczematous canal
Give Topical combination steroid and antibiotic drops s
Swab
may need referral for micro-suction.

If a patient fails to respond to topical antibiotics then the patient should be referred to ENT.

How well did you know this?
1
Not at all
2
3
4
5
Perfectly
41
Q

Anterior uveitis presentation

A

Features
acute onset
ocular discomfort & pain (may increase with use)
pupil may be irregular and small
photophobia (often intense)
blurred vision
red eye
lacrimation
ciliary flush
hypopyon; describes pus and inflammatory cells in the anterior chamber, often resulting in a visible fluid level
visual acuity initially normal → impaired

Associated conditions
ankylosing spondylitis
reactive arthritis
ulcerative colitis, Crohn's disease
Behcet's disease
sarcoidosis: bilateral disease may be seen
How well did you know this?
1
Not at all
2
3
4
5
Perfectly
42
Q

Anterior uveitis management

A

Management
urgent review by ophthalmology
cycloplegics (dilates the pupil which helps to relieve pain and photophobia) e.g. Atropine, cyclopentolate
steroid eye drops

How well did you know this?
1
Not at all
2
3
4
5
Perfectly
43
Q

Subacute thyroiditis

A

occur following viral infection and typically presents with hyperthyroidism.

There are typically 4 phases;
phase 1 (lasts 3-6 weeks): hyperthyroidism, painful goitre, raised ESR
phase 2 (1-3 weeks): euthyroid
phase 3 (weeks - months): hypothyroidism
phase 4: thyroid structure and function goes back to normal

Investigations
thyroid scintigraphy: globally reduced uptake of iodine-131

Management
usually self-limiting - most patients do not require treatment
thyroid pain may respond to aspirin or other NSAIDs
in more severe cases steroids are used, particularly if hypothyroidism develops

How well did you know this?
1
Not at all
2
3
4
5
Perfectly
44
Q

Pulsus paradoxus

A

greater than the normal (10 mmHg) fall in systolic blood pressure during inspiration → faint or absent pulse in inspiration

  • severe asthma, cardiac tamponade
How well did you know this?
1
Not at all
2
3
4
5
Perfectly
45
Q

Slow-rising/plateau

A

aortic stenosis

How well did you know this?
1
Not at all
2
3
4
5
Perfectly
46
Q

Pulsus alternans

A

regular alternation of the force of the arterial pulse

severe LVF

How well did you know this?
1
Not at all
2
3
4
5
Perfectly
47
Q

Bisferiens pulse

A

‘double pulse’ - two systolic peaks

mixed aortic valve disease

How well did you know this?
1
Not at all
2
3
4
5
Perfectly
48
Q

‘Jerky’ pulse

A

hypertrophic obstructive cardiomyopathy*

How well did you know this?
1
Not at all
2
3
4
5
Perfectly
49
Q

Legionnaire’s disease

A

Legionella pneumophilia. It typically colonizes water tanks and hence questions may hint at air-conditioning systems or foreign holidays. Person-to-person transmission is not seen

Features
flu-like symptoms including fever (present in > 95% of patients)
dry cough
relative bradycardia
confusion
lymphopaenia
hyponatraemia
deranged liver function tests
pleural effusion: seen in around 30% of patients

iagnosis
urinary antigen

Management
treat with erythromycin/clarithromycin

How well did you know this?
1
Not at all
2
3
4
5
Perfectly
50
Q

Superficial spreading melanoma

A

70% of cases
Arms, legs, back and chest, young people
A growing moles
flat patch of pigmented skin which grows slowly, often taking months or years to be recognised. It can be recognised by an ABCDE approach (asymmetry, border irregularity, colour variation, diameter >6mm and evolving)

How well did you know this?
1
Not at all
2
3
4
5
Perfectly
51
Q

routine manintance fluids adult

A

NICE guidelines from 2013 state for routine maintenance fluids the water requirement is approximately 25-30 ml/kg/day.

How well did you know this?
1
Not at all
2
3
4
5
Perfectly
52
Q

Osteogenesis imperfecta
Features
Investigations

A

Overview
autosomal dominant
abnormality in type 1 collagen due to decreased synthesis of pro-alpha 1 or pro-alpha 2 collagen polypeptides

Features
presents in childhood
fractures following minor trauma
blue sclera
deafness secondary to otosclerosis
dental imperfections are common

Investigations
adjusted calcium, phosphate, parathyroid hormone and ALP results are usually normal in osteogenesis imperfecta

How well did you know this?
1
Not at all
2
3
4
5
Perfectly
53
Q

Smudge or smear cells

A

CLL

How well did you know this?
1
Not at all
2
3
4
5
Perfectly
54
Q

Reed-Sternberg cells

A

Hodgkin’s lymphoma.

How well did you know this?
1
Not at all
2
3
4
5
Perfectly
55
Q

Rouleaux formation

A

multiple myeloma.

How well did you know this?
1
Not at all
2
3
4
5
Perfectly
56
Q

Auer rods

A

a feature of AML.

How well did you know this?
1
Not at all
2
3
4
5
Perfectly
57
Q

Pseudo Pelger-Huet cells

A

CML

How well did you know this?
1
Not at all
2
3
4
5
Perfectly
58
Q

Hypothermia on ECG

A

J Wave

How well did you know this?
1
Not at all
2
3
4
5
Perfectly
59
Q

reverse tick sign ECG

A

dioxin

How well did you know this?
1
Not at all
2
3
4
5
Perfectly
60
Q

PE management

A

the use of direct oral anticoagulants (DOACs) as first-line treatment for most people with VTE
the use of DOACs in patients with active cancer, as opposed to low-molecular weight heparin as was the previous recommendation
outpatient treatment in low-risk pulmonary embolism (PE) patients
routine cancer screening is no longer recommended following a VTE diagnosis

if neither apixaban or rivaroxaban are suitable then either LMWH followed by dabigatran or edoxaban OR LMWH followed by a vitamin K antagonist (VKA, i.e. warfarin)
if the patient has active cancer
previously LMWH was recommended
the new guidelines now recommend using a DOAC, unless this is contraindicated

Anticoag for at least three months if provoked

Thrombolysis
thrombolysis is now recommended as the first-line treatment for massive PE where there is circulatory failure (e.g. hypotension)
other invasive approaches should be considered where appropriate facilities exist

How well did you know this?
1
Not at all
2
3
4
5
Perfectly
61
Q

Toxic epidermal necrolysis

A
Drugs known to induce TEN
phenytoin
sulphonamides
allopurinol
penicillins
carbamazepine
NSAIDs
How well did you know this?
1
Not at all
2
3
4
5
Perfectly
62
Q

Pericarditis Features

A

Features
chest pain: may be pleuritic. Is often relieved by sitting forwards
other symptoms include non-productive cough, dyspnoea and flu-like symptoms
pericardial rub
tachypnoea
tachycardia

How well did you know this?
1
Not at all
2
3
4
5
Perfectly
63
Q

Pericarditis causes

A
Causes
viral infections (Coxsackie)
tuberculosis
uraemia (causes 'fibrinous' pericarditis)
trauma
post-myocardial infarction, Dressler's syndrome
connective tissue disease
hypothyroidism
malignancy
How well did you know this?
1
Not at all
2
3
4
5
Perfectly
64
Q

Pericarditis Investigations & Management

A

Investigations
ECG changes
the changes in pericarditis are often global/widespread, as opposed to the ‘territories’ seen in ischaemic events
‘saddle-shaped’ ST elevation
PR depression: most specific ECG marker for pericarditis
all patients with suspected acute pericarditis should have transthoracic echocardiography

Management
treat the underlying cause
a combination of NSAIDs and colchicine is now generally used for first-line for patients with acute idiopathic or viral pericarditis

How well did you know this?
1
Not at all
2
3
4
5
Perfectly
65
Q

Type 1 RTA

A

distal)
inability to generate acid urine (secrete H+) in distal tubule
causes hypokalaemia
complications include nephrocalcinosis and renal stones
causes include idiopathic, rheumatoid arthritis, SLE, Sjogren’s, amphotericin B toxicity, analgesic nephropathy

How well did you know this?
1
Not at all
2
3
4
5
Perfectly
66
Q

absolute contraindication to the use of regional anaesthesia such as spinal, epidural or plexus block?

A

Therapeutic anticoagulation is an absolute contraindication to the use of regional anaesthesia due to the risk of bleeding and the severity of a hematoma within the rigid space of the central nervous system.

How well did you know this?
1
Not at all
2
3
4
5
Perfectly
67
Q

Ankylosing spondylitis

A

Features
typically a young man who presents with lower back pain and stiffness of insidious onset
stiffness is usually worse in the morning and improves with exercise
the patient may experience pain at night which improves on getting up

Clinical examination
reduced lateral flexion
reduced forward flexion - Schober’s test - a line is drawn 10 cm above and 5 cm below the back dimples (dimples of Venus). The distance between the two lines should increase by more than 5 cm when the patient bends as far forward as possible
reduced chest expansion

How well did you know this?
1
Not at all
2
3
4
5
Perfectly
68
Q

lithium toxciity precipitated by

A

Toxicity may be precipitated by:
dehydration
renal failure
drugs: diuretics (especially thiazides), ACE inhibitors/angiotensin II receptor blockers, NSAIDs and metronidazole.

How well did you know this?
1
Not at all
2
3
4
5
Perfectly
69
Q

Diclofenac NSAID - contraindications

A

Diclofenac is now contraindicated with any form of cardiovascular disease

How well did you know this?
1
Not at all
2
3
4
5
Perfectly
70
Q

Degenerative cervical myelopathy

A

DCM symptoms can include any combination of [1]:
Pain (affecting the neck, upper or lower limbs)
Loss of motor function (loss of digital dexterity, preventing simple tasks such as holding a fork or doing up their shirt buttons, arm or leg weakness/stiffness leading to impaired gait and imbalance
Loss of sensory function causing numbness
Loss of autonomic function (urinary or faecal incontinence and/or impotence) - these can occur and do not necessarily suggest cauda equina syndrome in the absence of other hallmarks of that condition
Hoffman’s sign: is a reflex test to assess for cervical myelopathy. It is performed by gently flicking one finger on a patient’s hand. A positive test results in reflex twitching of the other fingers on the same hand in response to the flick.

(GOLD Standard - MRI of cervical spine if cervical suspected )

How well did you know this?
1
Not at all
2
3
4
5
Perfectly
71
Q

primary hyperparathyroidism

features

A

Features - ‘bones, stones, abdominal groans and psychic moans’
polydipsia, polyuria
peptic ulceration/constipation/pancreatitis
bone pain/fracture
renal stones
depression
hypertension

How well did you know this?
1
Not at all
2
3
4
5
Perfectly
72
Q

primary hyperparathyroidism

investigations

A

Investigations
raised calcium, low phosphate
PTH may be raised or (inappropriately, given the raised calcium) normal
technetium-MIBI subtraction scan
pepperpot skull is a characteristic X-ray finding of hyperparathyroidism

Treatment
the definitive management is total parathyroidectomy
conservative management may be offered if the calcium level is less than 0.25 mmol/L above the upper limit of normal AND the patient is > 50 years AND there is no evidence of end-organ damage
calcimimetic agents such as cinacalcet are sometimes used in patients who are unsuitable for surgery

How well did you know this?
1
Not at all
2
3
4
5
Perfectly
73
Q

Ottawa Rules with for ankle x-rays

A

The Ottawa Rules with for ankle x-rays have a sensitivity approaching 100%

An ankle x-ray is required only if there is any pain in the malleolar zone and any one of the following findings:
bony tenderness at the lateral malleolar zone (from the tip of the lateral malleolus to include the lower 6 cm of posterior border of the fibular)
bony tenderness at the medial malleolar zone (from the tip of the medial malleolus to the lower 6 cm of the posterior border of the tibia)
inability to walk four weight bearing steps immediately after the injury and in the emergency department

How well did you know this?
1
Not at all
2
3
4
5
Perfectly
74
Q

New York Heart Association (NYHA) classification

A

NYHA Class I
no symptoms
no limitation: ordinary physical exercise does not cause undue fatigue, dyspnoea or palpitations

NYHA Class II
mild symptoms
slight limitation of physical activity: comfortable at rest but ordinary activity results in fatigue, palpitations or dyspnoea

NYHA Class III
moderate symptoms
marked limitation of physical activity: comfortable at rest but less than ordinary activity results in symptoms

NYHA Class IV
severe symptoms
unable to carry out any physical activity without discomfort: symptoms of heart failure are present even at rest with increased discomfort with any physical activity

How well did you know this?
1
Not at all
2
3
4
5
Perfectly
75
Q

DDX breathing difficulty in children

A
Infection
(ie croup, URTI) 
If stridor present - Larynomalcia, asthma
Allergy 
Cardiac (malformation in young children)
How well did you know this?
1
Not at all
2
3
4
5
Perfectly
76
Q

croup

presentation

A

Parainfluenza viruses

Features
stridor
barking cough (worse at night)
fever
coryzal symptoms
How well did you know this?
1
Not at all
2
3
4
5
Perfectly
77
Q

croup

when to admit

A

CKS suggest admitting any child with moderate or severe croup. Other features which should prompt admission include:
< 6 months of age
known upper airway abnormalities (e.g. Laryngomalacia, Down’s syndrome)
uncertainty about diagnosis (important differentials include acute epiglottitis, bacterial tracheitis, peritonsillar abscess and foreign body inhalation)

How well did you know this?
1
Not at all
2
3
4
5
Perfectly
78
Q

croup investigations

A

the vast majority of children are diagnosed clinically
however, if a chest x-ray is done:
a posterior-anterior view will show subglottic narrowing, commonly called the ‘steeple sign’
in contrast, a lateral view in acute epiglottis will show swelling of the epiglottis - the ‘thumb sign’

How well did you know this?
1
Not at all
2
3
4
5
Perfectly
79
Q

croup management

A

Management
CKS recommend giving a single dose of oral dexamethasone (0.15mg/kg) to all children regardless of severity
prednisolone is an alternative if dexamethasone is not available

Emergency treatment
high-flow oxygen
nebulised adrenaline

How well did you know this?
1
Not at all
2
3
4
5
Perfectly
80
Q

Acute epiglottitis

A

Haemophilus influenzae type B.

Prompt recognition and treatment is essential as airway obstruction may develop. Epiglottitis generally occurs in children between the ages of 2 and 6 years. The incidence of epiglottitis has decreased since the introduction of the Hib vaccine

Features
rapid onset
unwell, toxic child
stridor
drooling of saliva
How well did you know this?
1
Not at all
2
3
4
5
Perfectly
81
Q

Acute epiglottitis

management

A

Management
immediate senior involvement, including those able to provide emergency airway support (e.g. anaesthetics, ENT)
endotracheal intubation may be necessary to protect the airway
if suspected do NOT examine the throat due to the risk of acute airway obstruction
the diagnosis is made by direct visualisation but this should only be done by senior staff who are able to intubate if necessary
oxygen
intravenous antibiotics

How well did you know this?
1
Not at all
2
3
4
5
Perfectly
82
Q

Parainfluenza virus

A

croup

How well did you know this?
1
Not at all
2
3
4
5
Perfectly
83
Q

Rhinovirus

A

cold

How well did you know this?
1
Not at all
2
3
4
5
Perfectly
84
Q

Streptococcus pneumoniae

A

he most common cause of community-acquired pneumonia

How well did you know this?
1
Not at all
2
3
4
5
Perfectly
85
Q

Haemophilus influenzae

A

Community-acquired pneumonia
Most common cause of bronchiectasis exacerbations
Acute epiglottitis

How well did you know this?
1
Not at all
2
3
4
5
Perfectly
86
Q

Staphylococcus aureus

A

Pneumonia, particularly following influenza

How well did you know this?
1
Not at all
2
3
4
5
Perfectly
87
Q

Mycoplasma pneumoniae

A

Atypical pneumonia

Flu-like symptoms classically precede a dry cough. Complications include haemolytic anaemia and erythema multiforme

How well did you know this?
1
Not at all
2
3
4
5
Perfectly
88
Q

Pneumocystis jiroveci

A

Common cause of pneumonia in HIV patients. Typically patients have few chest signs and develop exertional dyspnoea

How well did you know this?
1
Not at all
2
3
4
5
Perfectly
89
Q

Mycobacterium tuberculosis

A

Causes tuberculosis. A wide range of presentations from asymptomatic to disseminated disease are possible. Cough, night sweats and weight loss may be seen

How well did you know this?
1
Not at all
2
3
4
5
Perfectly
90
Q

Wheeze questions

A
  • Infection - systemic illness
  • Associated symptoms
    -Associated with excersies
    Associated with environment (pet, dust, smoking, mould, damp,
    _ Imms
How well did you know this?
1
Not at all
2
3
4
5
Perfectly
91
Q

Ddx Cyanosis

A
  • Illness and infection
  • cardiac (TOF, ebsteins, tricuspid atresia, transposition)

Medications (quinones)

Resp - choanal atresia, Pierre Robin Sequence, airway hemangioma, vascular rings/slings, cystic hygroma, and micrognathia

Blood related disorders - Polycythemia, Methemoglobinemia

How well did you know this?
1
Not at all
2
3
4
5
Perfectly
92
Q

pansystolic murmur at his lower left sternal edge and a loud P2.

A

Ventricular septal defect

How well did you know this?
1
Not at all
2
3
4
5
Perfectly
93
Q

typical features of heart failure in child

A

breathlessness, difficulty feeding, poor growth at an early age

How well did you know this?
1
Not at all
2
3
4
5
Perfectly
94
Q

VSD presentation

associations

A

classically a pan-systolic murmur which is louder in smaller defects

chromosomal disorders (e.g. Down’s syndrome, Edward’s syndrome, Patau syndrome) and single gene disorders such as Non-congenital causes include post myocardial infarction

How well did you know this?
1
Not at all
2
3
4
5
Perfectly
95
Q

VSD complications

A
Complications
aortic regurgitation*
infective endocarditis
Eisenmenger's complex
right heart failure
pulmonary hypertension: pregnancy is contraindicated in women with pulmonary hypertension as it carries a 30-50% risk of mortality
How well did you know this?
1
Not at all
2
3
4
5
Perfectly
96
Q

ostium secundum

A

stium secundum (70% of ASDs)
associated with Holt-Oram syndrome (tri-phalangeal thumbs)
ECG: RBBB with RAD

How well did you know this?
1
Not at all
2
3
4
5
Perfectly
97
Q

Ostium primum

A

present earlier than ostium secundum defects
associated with abnormal AV valves
ECG: RBBB with LAD, prolonged PR interval

How well did you know this?
1
Not at all
2
3
4
5
Perfectly
98
Q

Coarctation of the aorta presentation

A

infancy: heart failure
adult: hypertension
radio-femoral delay
mid systolic murmur, maximal over back
apical click from the aortic valve
notching of the inferior border of the ribs (due to collateral vessels) is not seen in young children

How well did you know this?
1
Not at all
2
3
4
5
Perfectly
99
Q

Coarctation of the aorta associations

A
Associations
Turner's syndrome
bicuspid aortic valve
berry aneurysms
neurofibromatosis
How well did you know this?
1
Not at all
2
3
4
5
Perfectly
100
Q

hypertrophic obstructive cardiomyopathy associated pulse

A

bisferiens pulse (double pulse).

How well did you know this?
1
Not at all
2
3
4
5
Perfectly
101
Q

Aortic stenosis management

A

VR if symptomatic, otherwise cut-off is gradient of 40 mmHg

How well did you know this?
1
Not at all
2
3
4
5
Perfectly
102
Q

Features of severe aortic stenosis

A
narrow pulse pressure
slow rising pulse
delayed ESM
soft/absent S2
S4
thrill
duration of murmur
left ventricular hypertrophy or failure
How well did you know this?
1
Not at all
2
3
4
5
Perfectly
103
Q

Aortic regurgitation features

A

ifficulty breathing at night, occasional palpitations and tight chest pain. On examination, he has a collapsing pulse and a laterally shifted apex beat
murmur = early diastolic

How well did you know this?
1
Not at all
2
3
4
5
Perfectly
104
Q

Abdominal pain ddx in children

A
  • infection (Gastroenteritis)
    -Constipation
    _ Appendcitis
    Merkles adentisi
  • Hernia
    Intussception
    Allergery
    Hischsprung
    IBS
    coalic
    -
How well did you know this?
1
Not at all
2
3
4
5
Perfectly
105
Q

Attention Deficit Hyperactivity Disorder presentaion

A
  • inattentiveness
    hyperactivity and impulsiveness
    associated mood or behaviour disorders
How well did you know this?
1
Not at all
2
3
4
5
Perfectly
106
Q

Attention Deficit Hyperactivity Disorder management

A

watch and wait

(CAMHS ref needed)

Drug therapy should be seen as a last resort and is only available to those aged 5 years or more

Methylphenidate is first line in children

f there is inadequate response, switch to lisdexamfetamine;

How well did you know this?
1
Not at all
2
3
4
5
Perfectly
107
Q

Methylphenidate side effects

A

ide-effects include abdominal pain, nausea and dyspepsia. In children, weight and height should be monitored every 6 months

How well did you know this?
1
Not at all
2
3
4
5
Perfectly
108
Q

Features Bronchiolitis

A

eatures
coryzal symptoms (including mild fever) precede:
dry cough
increasing breathlessness
wheezing, fine inspiratory crackles (not always present)
feeding difficulties associated with increasing dyspnoea are often the reason for hospital admission

How well did you know this?
1
Not at all
2
3
4
5
Perfectly
109
Q

Bronchiolitis - NICE recommend immediate referral (usually by 999 ambulance) if they have any of the following:

A

apnoea (observed or reported)
child looks seriously unwell to a healthcare professional
severe respiratory distress, for example grunting, marked chest recession, or a respiratory rate of over 70 breaths/minute
central cyanosis
persistent oxygen saturation of less than 92% when breathing air.

How well did you know this?
1
Not at all
2
3
4
5
Perfectly
110
Q

Bronchiolitis- management

A

Management is largely supportive
humidified oxygen is given via a head box and is typically recommended if the oxygen saturations are persistently < 92%
nasogastric feeding may be needed if children cannot take enough fluid/feed by mouth
suction is sometimes used for excessive upper airway secretions

How well did you know this?
1
Not at all
2
3
4
5
Perfectly
111
Q

causes Cerebral palsy

A

Causes
antenatal (80%): e.g. cerebral malformation and congenital infection (rubella, toxoplasmosis, CMV)
intrapartum (10%): birth asphyxia/trauma
postnatal (10%): intraventricular haemorrhage, meningitis, head-trauma

How well did you know this?
1
Not at all
2
3
4
5
Perfectly
112
Q

presentations Cerebral palsy

A
Possible manifestations include:
abnormal tone early infancy
delayed motor milestones
abnormal gait
feeding difficultie.
How well did you know this?
1
Not at all
2
3
4
5
Perfectly
113
Q

Cerebral palsy

A

Management
as with any child with a chronic condition a multidisciplinary approach is needed
treatments for spasticity include oral diazepam, oral and intrathecal baclofen, botulinum toxin type A, orthopaedic surgery and selective dorsal rhizotomy
anticonvulsants, analgesia as required

How well did you know this?
1
Not at all
2
3
4
5
Perfectly
114
Q

Exanthems

A

eruptive skin rashes associated with a fever or other constitutional symptoms

How well did you know this?
1
Not at all
2
3
4
5
Perfectly
115
Q

Causes of maculopapular eruptions

A
Measles (rubeola)
Rubella
Erythema infectiosum (slapped cheek)
Exanthum subitum (roseola)
Lyme disease
Pityriasis
Drug-related eruptions
Erythema multiforme
How well did you know this?
1
Not at all
2
3
4
5
Perfectly
116
Q

Causes of diffuse erythema with desquamation

A

Scarlet fever
Toxic shock syndrome
Staphylococcal scaled skin syndrome
Kawasaki disease

How well did you know this?
1
Not at all
2
3
4
5
Perfectly
117
Q

Measles rash

A

erythematous maculopapular rash beginning on the head, with a cephalocaudad progression.

Blue-white Kopliks spots occurring on the buccal mucosa opposite the second molar and lasting around 1-2 day

How well did you know this?
1
Not at all
2
3
4
5
Perfectly
118
Q

Measles management

A

Measles is a notifiable illness and notification is required based on clinical suspicion.

Children diagnosed with measles should be kept off school until 5 days after the appearance of the rash.

The treatment of measles is largely symptomatic and the majority of children will recover uneventfully.

How well did you know this?
1
Not at all
2
3
4
5
Perfectly
119
Q

Meckel’s diverticulum

presentation

A

Presentation (usually asymptomatic)
abdominal pain mimicking appendicitis
rectal bleeding
Meckel’s diverticulum is the most common cause of painless massive GI bleeding requiring a transfusion in children between the ages of 1 and 2 years
intestinal obstruction
secondary to an omphalomesenteric band (most commonly), volvulus and intussusception

How well did you know this?
1
Not at all
2
3
4
5
Perfectly
120
Q

Anaphylaxis presentation

A

Anaphylaxis may be defined as a severe, life-threatening, generalised or systemic hypersensitivity reaction.

Common identified causes of anaphylaxis:
food (e.g. nuts) - the most common cause in children
drugs
venom (e.g. wasp sting)

How well did you know this?
1
Not at all
2
3
4
5
Perfectly
121
Q

Anaphylaxis management

A

Adrenaline is by far the most important drug in anaphylaxis and should be given as soon as possible.

also give Hydrocortisone and Chlorphenamine

Adrenaline can be repeated every 5 minutes if necessary. The best site for IM injection is the anterolateral aspect of the middle third of the thigh.

Management following stabilisation:
patients who have had emergency treatment for anaphylaxis should be observed for 6–12 hours from the onset of symptoms, as it is known that biphasic reactions can occur in up to 20% of patients
sometimes it can be difficult to establish whether a patient had a true episode of anaphylaxis. Serum tryptase levels are sometimes taken in such patients as they remain elevated for up to 12 hours following an acute episode of anaphylaxis.v

How well did you know this?
1
Not at all
2
3
4
5
Perfectly
122
Q

Anaphylaxis test after episode to confirm

A

serum tryptase levels rise following an acute episode

How well did you know this?
1
Not at all
2
3
4
5
Perfectly
123
Q

Acute exacerbations of COPD features

A

Features
increase in dyspnoea, cough, wheeze
there may be an increase in sputum suggestive of an infective cause
patients may be hypoxic and in some cases have acute confusion

How well did you know this?
1
Not at all
2
3
4
5
Perfectly
124
Q

The most common bacterial organisms that cause infective exacerbations of COPD are:

A

Haemophilus influenzae (most common cause)
Streptococcus pneumoniae
Moraxella catarrhalis

How well did you know this?
1
Not at all
2
3
4
5
Perfectly
125
Q

Infective exacerbations of COPD managements

A

NICE guidelines from 2010 recommend the following:
increase frequency of bronchodilator use and consider giving via a nebuliser
give prednisolone 30 mg daily for 5 days
it is common practice for all patients with an exacerbation of COPD to receive antibiotics. NICE do not support this approach. They recommend giving oral antibiotics ‘if sputum is purulent or there are clinical signs of pneumonia’
the BNF recommends one of the following oral antibiotics first-line: amoxicillin or clarithromycin or doxycycline.

How well did you know this?
1
Not at all
2
3
4
5
Perfectly
126
Q

pneumonia causes

A

bacterial pneumonia is by far the most common type of pneumonia
Streptococcus pneumoniae = most common
Haemophilus influenzae = COPD
Staphylococcus aureus = occurs in patient following influenza infection
Klebsiella pneumoniae = alcoholics

How well did you know this?
1
Not at all
2
3
4
5
Perfectly
127
Q

Pneumonia Bloods

A

Bloods
full blood count: would usually show a neutrophilia in bacterial infections
urea and electrolytes: check for dehydration (remember the ‘U’ for urea in CURB-65, see below) and also other changes seen with some atypical pneumonias
CRP: raised in response to infection

How well did you know this?
1
Not at all
2
3
4
5
Perfectly
128
Q

Pneumonia management

A

Patients with pneumonia require the following:
antibiotics: to treat the underlying infection
supportive care: for example oxygen therapy if the patients is hypoxaemic, intravenous fluids if the patient is hypotensive or shows signs of dehydration

CURB- 65 score

How well did you know this?
1
Not at all
2
3
4
5
Perfectly
129
Q

Pneumonia - Discharge criteria and advice post-discharge

A

Discharge criteria and advice post-discharge

NICE recommend that patients are not routinely discharged if in the past 24 hours they have had 2 or more of the following findings:
temperature higher than 37.5°C
respiratory rate 24 breaths per minute or more
heart rate over 100 beats per minute
systolic blood pressure 90 mmHg or less
oxygen saturation under 90% on room air
abnormal mental status
inability to eat without assistance.

They also recommend delaying discharge if the temperature is higher than 37.5°C.

How well did you know this?
1
Not at all
2
3
4
5
Perfectly
130
Q

Pneumothorax risk factors

A

Risk factors
pre-existing lung disease: COPD, asthma, cystic fibrosis, lung cancer, Pneumocystis pneumonia
connective tissue disease: Marfan’s syndrome, rheumatoid arthritis
ventilation, including non-invasive ventilation
catamenial pneumothorax is the cause of 3-6% of spontaneous pneumothoraces occurring in menstruating women. It is thought to be caused by endometriosis within the thorax

How well did you know this?
1
Not at all
2
3
4
5
Perfectly
131
Q

Pneumothorax symptoms

A
Symptoms tend to come on suddenly. Features include:
dyspnoea
chest pain: often pleuritic
sweating
tachypnoea
tachycardia
How well did you know this?
1
Not at all
2
3
4
5
Perfectly
132
Q

Primary pneumothorax

management

A

Recommendations include:
if the rim of air is < 2cm and the patient is not short of breath then discharge should be considered
otherwise, aspiration should be attempted
if this fails (defined as > 2 cm or still short of breath) then a chest drain should be inserted
patients should be advised to avoid smoking to reduce the risk of further episodes - the lifetime risk of developing a pneumothorax in healthy smoking men is around 10% compared with around 0.1% in non-smoking men

How well did you know this?
1
Not at all
2
3
4
5
Perfectly
133
Q

Secondary pneumothorax management

A

Recommendations include:
if the patient is > 50 years old and the rim of air is > 2cm and/or the patient is short of breath then a chest drain should be inserted.
otherwise aspiration should be attempted if the rim of air is between 1-2cm. If aspiration fails (i.e. pneumothorax is still greater then 1cm) a chest drain should be inserted. All patients should be admitted for at least 24 hours
if the pneumothorax is less the 1cm then the BTS guidelines suggest giving oxygen and admitting for 24 hours
regarding scuba diving, the BTS guidelines state: ‘Diving should be permanently avoided unless the patient has undergone bilateral surgical pleurectomy and has normal lung function and chest CT scan postoperatively.’

How well did you know this?
1
Not at all
2
3
4
5
Perfectly
134
Q

triad of PE

and clinical signs

A

pleuritic chest pain, dyspnoea and haemoptysis.

The relative frequency of common clinical signs is shown below:
Tachypnea (respiratory rate >20/min) - 96%
Crackles - 58%
Tachycardia (heart rate >100/min) - 44%
Fever (temperature >37.8°C) - 43%

How well did you know this?
1
Not at all
2
3
4
5
Perfectly
135
Q

PE suspected

A

If a PE is suspected a 2-level PE Wells score should be performed:

linical probability simplified scores
PE likely - more than 4 points
PE unlikely - 4 points or less

If a PE is ‘likely’ (more than 4 points) arrange an immediate computed tomography pulmonary angiogram (CTPA). If there is a delay in getting the CTPA then interim therapeutic anticoagulation should be given until the scan is performed.
interim therapeutic anticoagulation used to mean giving low-molecular weight heparin
NICE updated their guidance in 2020. They now recommend using an anticoagulant that can be continued if the result is positive.
this means normally a direct oral anticoagulant (DOAC) such as apixaban or rivaroxaban

How well did you know this?
1
Not at all
2
3
4
5
Perfectly
136
Q

test in PE

A

CTPA is now the recommended initial lung-imaging modality for non-massive PE. Advantages compared to V/Q scans include speed, easier to perform out-of-hours, a reduced need for further imaging and the possibility of providing an alternative diagnosis if PE is excluded
if the CTPA is negative then patients do not need further investigations or treatment for PE
V/Q scanning may be used initially if appropriate facilities exist, the chest x-ray is normal, and there is no significant symptomatic concurrent cardiopulmonary disease. V/Q scanning is also the investigation of choice if there is renal impairment (doesn’t require the use of contrast unlike CTPA)

How well did you know this?
1
Not at all
2
3
4
5
Perfectly
137
Q

Pulmonary embolism and renal impairment →

A

V/Q scan is the investigation of choice

How well did you know this?
1
Not at all
2
3
4
5
Perfectly
138
Q

Massive PE + hypotension

A
  • thrombolyse
How well did you know this?
1
Not at all
2
3
4
5
Perfectly
139
Q

bloods PE

A

Pulmonary embolism causes hyperventilation, causing a drop in arterial carbonic dioxide partial pressure and thus alkalosis.

How well did you know this?
1
Not at all
2
3
4
5
Perfectly
140
Q

Features of pulmonary oedema on a chest x-ray may include:

A

Features of pulmonary oedema on a chest x-ray may include:
interstitial oedema
bat’s wing appearance
upper lobe diversion (increased blood flow to the superior parts of the lung)
Kerley B lines
pleural effusion
cardiomegaly may be seen if there is cardiogenic cause

How well did you know this?
1
Not at all
2
3
4
5
Perfectly
141
Q

Tricuspid regurgitation signs

A

Pan systolic mumur
JVP increase
Hepatomegaly
Heave

How well did you know this?
1
Not at all
2
3
4
5
Perfectly
142
Q

Tricuspid regurgitation causes

A

Right V INFARCT

Pulmonary hypertension

Rheumatic heart disease

Infective endo

Ébstekns

How well did you know this?
1
Not at all
2
3
4
5
Perfectly
143
Q

NSTEMI

A

All patients should receive
aspirin 300mg
nitrates or morphine to relieve chest pain if required
Clopidogrel

How well did you know this?
1
Not at all
2
3
4
5
Perfectly
144
Q

optic neuritis causes

A

Causes
multiple sclerosis
diabetes
syphilis

How well did you know this?
1
Not at all
2
3
4
5
Perfectly
145
Q

optic neuritis features

A

Features
unilateral decrease in visual acuity over hours or days
poor discrimination of colours, ‘red desaturation’
pain worse on eye movement
relative afferent pupillary defect
central scotoma

How well did you know this?
1
Not at all
2
3
4
5
Perfectly
146
Q

optic neuritis management

A

Management
high-dose steroids
recovery usually takes 4-6 weeks

Prognosis
MRI: if > 3 white-matter lesions, 5-year risk of developing multiple sclerosis is c. 50%

How well did you know this?
1
Not at all
2
3
4
5
Perfectly
147
Q

abdo signs

A

A number of eponymous abdominal signs are noted. These include:
Rovsings sign- appendicitis
Boas sign -cholecystitis
Murphys sign- cholecystitis
Cullens sign- pancreatitis (other intraabdominal haemorrhage)
Grey-Turners sign- pancreatitis (or other retroperitoneal haemorrhage)

How well did you know this?
1
Not at all
2
3
4
5
Perfectly
148
Q

BV causes and treatment

A

Bacterial vaginosis (BV) describes an overgrowth of predominately anaerobic organisms such as Gardnerella vaginalis

Features
vaginal discharge: ‘fishy’, offensive
asymptomatic in 50%

Amsel’s criteria for diagnosis of BV - 3 of the following 4 points should be present
thin, white homogenous discharge
clue cells on microscopy: stippled vaginal epithelial cells
vaginal pH > 4.5
positive whiff test (addition of potassium hydroxide results in fishy odour)

Management
oral metronidazole for 5-7 days
70-80% initial cure rate
relapse rate > 50% within 3 months
the BNF suggests topical metronidazole or topical clindamycin as alternatives.
How well did you know this?
1
Not at all
2
3
4
5
Perfectly
149
Q

BV in pregnancy

A

Bacterial vaginosis in pregnancy
results in an increased risk of preterm labour, low birth weight and chorioamnionitis, late miscarriage
it was previously taught that oral metronidazole should be avoided in the first trimester and topical clindamycin used instead. Recent guidelines however recommend that oral metronidazole is used throughout pregnancy. The BNF still advises against the use of high dose metronidazole regimes

How well did you know this?
1
Not at all
2
3
4
5
Perfectly
150
Q

hypocalcaemia causes

A

auses
vitamin D deficiency (osteomalacia)
chronic kidney disease
hypoparathyroidism (e.g. post thyroid/parathyroid surgery)
pseudohypoparathyroidism (target cells insensitive to PTH)
rhabdomyolysis (initial stages)
magnesium deficiency (due to end organ PTH resistance)
massive blood transfusion
acute pancreatitis

How well did you know this?
1
Not at all
2
3
4
5
Perfectly
151
Q

hypocalcaemia Management

A

Management
acute management of severe hypocalcaemia is with intravenous replacement. The preferred method is with intravenous calcium gluconate, 10ml of 10% solution over 10 minutes
intravenous calcium chloride is more likely to cause local irritation
ECG monitoring is recommended
further management depends on the underlying cause

How well did you know this?
1
Not at all
2
3
4
5
Perfectly
152
Q

Bells palsy presentation

A

Features
lower motor neuron facial nerve palsy - forehead affected*
patients may also notice post-auricular pain (may precede paralysis), altered taste, dry eyes, hyperacusis

How well did you know this?
1
Not at all
2
3
4
5
Perfectly
153
Q

Bells palsy management

A

prednisolone 1mg/kg for 10 days should be prescribed for patients within 72 hours of onset of Bell’s palsy. Adding in aciclovir gives no additional benefit
eye care is important - prescription of artificial tears and eye lubricants should be considered

How well did you know this?
1
Not at all
2
3
4
5
Perfectly
154
Q

large fluid volumes risk

A

hyperchloraemic acidosis

How well did you know this?
1
Not at all
2
3
4
5
Perfectly
155
Q

stroke post coag

A

Aspirin 300 mg daily for 2 weeks should be given immediately after an ischaemic stroke is confirmed by brain imaging. Following this, clopidogrel 75 mg daily should be given long-term -if it can be tolerated and is not contraindicated.

If clopidogrel is contraindicated or not tolerated, then the patient should be given modified-release dipyridamole alongside low dose aspirin.

How well did you know this?
1
Not at all
2
3
4
5
Perfectly
156
Q

Age-related macular degeneration presentation

A

Patients typically present with a subacute onset of visual loss with:
a reduction in visual acuity, particularly for near field objects
difficulties in dark adaptation with an overall deterioration in vision at night
fluctuations in visual disturbance which may vary significantly from day to day
they may also suffer from photopsia, (a perception of flickering or flashing lights), and glare around objects

How well did you know this?
1
Not at all
2
3
4
5
Perfectly
157
Q

Age-related macular degeneration treatment

A

zinc with anti-oxidant vitamins A,C and E reduced progression of the disease by around one third.

anti-VEGF agents (wet) - instituted within the first two months of diagnosis of wet ARMD if possible.

Laser photocoagulation does slow progression of ARMD where there is new vessel formation, although there is a risk of acute visual loss after treatment, which may be increased in patients with sub-foveal ARMD. For this reason anti-VEGF therapies are usually preferred.

How well did you know this?
1
Not at all
2
3
4
5
Perfectly
158
Q

Central retinal vein occlusion presentation

A

Features
sudden, painless reduction or loss of visual acuity, usually unilaterally
severe retinal haemorrhages are usually seen on fundoscopy

How well did you know this?
1
Not at all
2
3
4
5
Perfectly
159
Q

Herpes simplex keratitis presentation

A
Features
red, painful eye
photophobia
epiphora
visual acuity may be decreased
fluorescein staining may show an epithelial ulcer
How well did you know this?
1
Not at all
2
3
4
5
Perfectly
160
Q

Herpes simplex keratitis management

A

Management
immediate referral to an ophthalmologist
topical aciclovir

How well did you know this?
1
Not at all
2
3
4
5
Perfectly
161
Q

Rentral retinal artery occlusion

A

causes sudden unilateral visual loss
due to thromboembolism (from atherosclerosis) or arteritis (e.g. temporal arteritis)
features include afferent pupillary defect, ‘cherry red’ spot on a pale retina

How well did you know this?
1
Not at all
2
3
4
5
Perfectly
162
Q

retinal detachment presentaition

A

Peripheral curtain over vision + spider webs + flashing lights in vision

(painless loss of vision)

How well did you know this?
1
Not at all
2
3
4
5
Perfectly
163
Q

amaurosis fugax

A

Painless, transient monocular blindness together with the description of a ‘black curtain coming down’

How well did you know this?
1
Not at all
2
3
4
5
Perfectly
164
Q

Tunnel vision is the concentric diminution of the visual fields

Causes

A
Causes
papilloedema
glaucoma
retinitis pigmentosa
choroidoretinitis
optic atrophy secondary to tabes dorsalis
hysteria
How well did you know this?
1
Not at all
2
3
4
5
Perfectly
165
Q

Acute angle closure glaucoma is associated with ?

A

hypermetropia,

How well did you know this?
1
Not at all
2
3
4
5
Perfectly
166
Q

primary open-angle glaucoma is associated with

A

myopia

How well did you know this?
1
Not at all
2
3
4
5
Perfectly
167
Q

Acyanotic heart disease

A

most common causes (blood contains oxygen)

ventricular septal defects (VSD) - most common, accounts for 30%
atrial septal defect (ASD)
patent ductus arteriosus (PDA)
coarctation of the aorta
aortic valve stenosis
How well did you know this?
1
Not at all
2
3
4
5
Perfectly
168
Q

Cyanotic heart disease

A

Cyanotic - most common causes
tetralogy of Fallot
transposition of the great arteries (TGA)
tricuspid atresia

169
Q

Anti-dsDNA

A

SLE

170
Q

Anti-CCP

A

RA

171
Q

Anti-La

A

Sjogren’s syndrome. It can also be found in patients with SLE, however, it is not very specific.

172
Q

ANCA

A

ANCA is an antibody targeted against neutrophils. It is seen in patients with autoimmune vasculitis.

173
Q

Anti-centromere

A

Anti-centromere antibodies are characteristic of the CREST syndrome which is the cutaneous form of systemic sclerosis. CREST stands for calcinosis, Raynaud’s, oesophageal dysmotility, sclerodactyly and telangiectasia.

174
Q

metoclopramide avoid when

A

bowel obstruction

as could cause a perforation in bowel obstruction.

175
Q

Osteochondritis dissecans presentation

A

Patients typically present with a subacute onset of:
Knee pain and swelling, typically after exercise
Knee catching, locking and/or giving way - more constant and severe symptoms are associated with the presence of loose bodies
Feeling a painful ‘clunk’ when flexing or extending the knee - indicating the involvement of the lateral femoral condyle

176
Q

Osteochondritis dissecans signs

A

Joint effusion
Full range of movement in the joint without signs of ligamentous instability
External tibial rotation when walking - if medial femoral involvement
Tenderness on palpation of the articular cartilage of the medial femoral condyle, when the knee is flexed
Wilson’s sign for detecting medial condyle lesion - with the knee at 90° flexion and tibia internally rotated, the gradual extension of the joint leads to pain at about 30°, external rotation of the tibia at this point relieves the pain

177
Q

Osteochondritis dissecans investigations

A

Investigations:
X-ray (anteroposterior, lateral and tunnel views) - may show the subchondral crescent sign or loose bodies
MRI - used to evaluate cartilage, visualise loose bodies, stage and assess the stability of the lesion
CT - may be used in preoperative planning and in cases where MRI is not available or contraindicated
Scintigraphy - may be used to guide treatment as it may show increased uptake in the fragments - a sign of osteoblastic activity

178
Q

treatments have not been shown to improve mortality in patients with chronic heart failure?

A

furosemide

179
Q

Heart failure management

A

The first-line treatment for all patients is both an ACE-inhibitor and a beta-blocker

Second-line treatment is an aldosterone antagonist

Third-line treatment should be initiated by a specialist. Options include ivabradine, sacubitril-valsartan, hydralazine in combination with nitrate, digoxin and cardiac resynchronisation therapy

180
Q

antibiotics for Exacerbations of chronic bronchitis

A

Amoxicillin or tetracycline or clarithromycin

181
Q

antibiotics for Uncomplicated community-acquired pneumonia

A

Amoxicillin (Doxycycline or clarithromycin in penicillin allergic, add flucloxacillin if staphylococci suspected e.g. In influenza)

182
Q

antibiotics for Pneumonia possibly caused by atypical pathogens

A

Clarithromycin

183
Q

antibiotics for Hospital-acquired pneumonia

A

Within 5 days of admission: co-amoxiclav or cefuroxime
More than 5 days after admission: piperacillin with tazobactam OR a broad-spectrum cephalosporin (e.g. ceftazidime) OR a quinolone (e.g. ciprofloxacin)

184
Q

Impetigo treatment

A

Topical hydrogen peroxide, oral flucloxacillin or erythromycin if widespread

185
Q

Cellulitis treatment

A

Flucloxacillin (clarithromycin, erythromycin or doxycycline if penicillin-allergic)

Co-amoxiclav (clarithromycin, + metronidazole if penicillin-allergic)

186
Q

Animal or human bite treat

A

Co-amoxiclav (doxycycline + metronidazole if penicillin-allergic)

187
Q

Salicylate poisoning presentation

A

present with nausea, vomiting, tinnitus and headache. In more severe overdoses, hyperventilation and a secondary respiratory alkalosis develops.

give IV sodium bicarb

188
Q

benzodiazepine overdose.

A

Flumazenil

189
Q

CLL is associated with

A

warm autoimmune haemolytic anaemia

190
Q

The Mackler triad for Boerhaave syndrome:

A

vomiting, thoracic pain, subcutaneous emphysema. It typically presents in middle aged men with a background of alcohol abuse.

= oesphgealo perforation

191
Q

Plummer-Vinson syndrome

A

Triad of:
dysphagia (secondary to oesophageal webs)
glossitis
iron-deficiency anaemia

Treatment includes iron supplementation and dilation of the webs

192
Q

Orlistat

A

weight loss

inhibiting gastric and pancreatic lipase to reduce the digestion of fat

193
Q

Features suggestive of hypernatraemic dehydration:

A
jittery movements
increased muscle tone
hyperreflexia
convulsions
drowsiness or coma
194
Q

X-linked recessive inheritance

A

males only effected
X-linked recessive disorders are transmitted by heterozygote females (carriers) and male-to-male transmission is not seen.
ie haemophilia A

195
Q

life treating asthma attack

A
SpO2 <92%
PEF <33% best or predicted
Silent chest
Poor respiratory effort
Agitation
Altered consciousness
Cyanosis
196
Q

risk factors for erectile dysfunction

A

Other than increasing age, risk factors include:
cardiovascular disease risk factors: obesity, diabetes mellitus, dyslipidaemia, metabolic syndrome, hypertension, smoking
alcohol use
drugs: SSRIs, beta-blockers

197
Q

assessment for erectile dysfunction

A

t all men have their 10-year cardiovascular risk calculated by measuring lipid and fasting glucose serum levels.

Free testosterone should also be measured in the morning between 9 and 11am. If free testosterone is low or borderline, it should be repeated along with follicle-stimulating hormone, luteinizing hormone and prolactin levels. If any of these are abnormal refer to endocrinology for further assessment.

198
Q

erectile dysfunction management

A

PDE-5 inhibitors (such as sildenafil, ‘Viagra’)

Vacuum erection devices are recommended as first-line treatment in those who can’t/won’t take a PDE-5 inhibitor.

Other points
for a young man who has always had difficulty achieving an erection, referral to urology is appropriate
people with erectile dysfunction who cycle for more than three hours per week should be advised to stop

199
Q

normocytic anaemia with low serum iron, low TIBC but raised ferritin

A

anaemia of chronic disease

200
Q

Sideroblastic anaemia

A

a microcytic anaemia with raised serum iron levels.

201
Q

a microcytic anaemia, low ferritin and a raised TIBC, not the current clinical picture.

A

iron def

202
Q

Subcapsular cataracts may be associated with what medication

A

steroids (Allopurinol less strong link )

203
Q

) Myotonic dystrophy is associated with what eye problem

A

dot cataracts

204
Q

Whipple’s triad of symptoms of for insluinoma

A

) hypoglycaemia with fasting or exercise, 2) reversal of symptoms with glucose, and 3) recorded low BMs at the time of symptoms is hallmark for an insulinoma

205
Q

insluinoma features

A

Features
of hypoglycaemia: typically early in morning or just before meal, e.g. diplopia, weakness etc
rapid weight gain may be seen
high insulin, raised proinsulin:insulin ratio
high C-peptide

206
Q

Rhabdomyolysis causes

A

Causes
seizure
collapse/coma (e.g. elderly patients collapses at home, found 8 hours later)
ecstasy
crush injury
McArdle’s syndrome
drugs: statins (especially if co-prescribed with clarithromycin)

207
Q

Hypothyroidism causes a euvolaemic hyponatraemia

blood results

A

Plasma osmolality low

NA in urine = high

208
Q

The recommended dose of adrenaline to give during advanced ALS

A

1mg

209
Q

Hodgkin’s lymphoma - best prognosis =

A

lymphocyte predominant

210
Q

Intramuscular ceftriaxone is the treatment of choice for

A

gonnorhoae

211
Q

Bifascicular block

A

combination of RBBB with left anterior or posterior hemiblock
e.g. RBBB with left axis deviation

212
Q

Trifascicular block

A

Trifascicular block

features of bifascicular block as above + 1st-degree heart block

213
Q

metformin not tolerated GI

A

If metformin is not tolerated due to GI side-effects, try a modified-release formulation before switching to a second-line agent

214
Q

Lithium monitoring

A

When checking lithium levels, the sample should be taken 12 hours post-dose

after starting lithium levels should be performed weekly and after each dose change until concentrations are stable

once established, lithium blood level should ‘normally’ be checked every 3 months

after a change in dose, lithium levels should be taken a week later and weekly until the levels are stable.

215
Q

Lithium SE

A

Adverse effects
nausea/vomiting, diarrhoea
fine tremor
nephrotoxicity: polyuria, secondary to nephrogenic diabetes insipidus
thyroid enlargement, may lead to hypothyroidism
ECG: T wave flattening/inversion
weight gain
idiopathic intracranial hypertension
leucocytosis
hyperparathyroidism and resultant hypercalcaemia

216
Q

GCS

A

Motor response

  1. Obeys commands
  2. Localises to pain
  3. Withdraws from pain
  4. Abnormal flexion to pain (decorticate posture)
  5. Extending to pain
  6. None

Verbal response

  1. Orientated
  2. Confused
  3. Words
  4. Sounds
  5. None

Eye opening

  1. Spontaneous
  2. To speech
  3. To pain
  4. None
217
Q

autoimmune hep - presentation

A

Features
may present with signs of chronic liver disease
acute hepatitis: fever, jaundice etc (only 25% present in this way)
amenorrhoea (common)
ANA/SMA/LKM1 antibodies, raised IgG levels
liver biopsy: inflammation extending beyond limiting plate ‘piecemeal necrosis’, bridging necrosis

218
Q

duloxetine

A

duloxetine. This acts as a serotonin/norepinephrine reuptake inhibitor and its common side effects include nausea, dizziness and insomnia.

219
Q

Lacunar strokes can present with

A

unilateral motor disturbance affecting the face, arm or leg or all 3.
complete one sided sensory loss.
ataxia hemiparesis.

220
Q

patient refuses IM ceftriaxone in gonorrhoea

A

oral cefixime + oral azithromycin

221
Q

optimal treatment in HNF1A-MODY.

A

Sulfonylureas (e.g. gliclazide)

222
Q

live attenuated vaccines which should not be given in immunocompromised patients are:

A
Yellow fever
Oral polio
Intranasal influenza
Varicella
Measles, mumps and rubella (MMR)
BCG
223
Q

most frequent malaria

A

Falciparum malaria is the commonest type of malaria

224
Q

recent parovirus in pregnancy immunology

Rubella

A

IgG positive & IgM negative - shows immunity to parvovirus. Reassure, no further action.
IgG negative & IgM positive - non-immune. Recent parvovirus infection in last 4 weeks. Refer immediately for further tests/fetal medicine.

IgG negative & IgM negative - repeat test in 4 weeks. If both tests still negative, this confirms susceptibility, but no recent infection. Reassure, further action required only if subsequent exposure occurs.

225
Q

Dupuytren’s contracture

A
Specific causes include:
manual labour
phenytoin treatment
alcoholic liver disease
diabetes mellitus
trauma to the hand
226
Q

Orbital cellulitis présentation

A
Presentation
Redness and swelling around the eye
Severe ocular pain
Visual disturbance
Proptosis
Ophthalmoplegia/pain with eye movements
Eyelid oedema and ptosis
Drowsiness +/- Nausea/vomiting in meningeal involvement (Rare)
227
Q

Orbital cellulitis investigations

A

nvestigations
Full blood count – WBC elevated, raised inflammatory markers.
Clinical examination involving complete ophthalmological assessment – Decreased vision, afferent pupillary defect, proptosis, dysmotility, oedema, erythema.
CT with contrast – Inflammation of the orbital tissues deep to the septum, sinusitis.
Blood culture and microbiological swab to determine the organism. Most common bacterial causes – Streptococcus, Staphylococcus aureus, Haemophilus influenzae B.

228
Q

Felty’s syndrome

A

RA + splenomegaly + low white cell count)

229
Q

Irradiated blood products when

A

Irradiated blood products are used to avoid transfusion-associated graft versus host disease

Neonates up to 28 days post expected date of delivery

Pregnancy: Elective transfusions during pregnancy (not during labour or delivery)

Immunocompromised (e.g. chemotherapy or congenital)

Patients with/previous Hodgkins Disease

HIV

230
Q

Injury to the AC joint - injury

A

AC joint injuries are graded I to VI depending on the degree of separation.

Grade I and II injuries are very common and are typically managed conservatively including resting the joint using a sling.

Grade IV, V and VI are rare and require surgical intervention.

The management of grade III injuries is a matter of debate and often depends on individual circumstances.

231
Q

sialadenitis

A

inflammation of the salivary gland likely secondary to obstruction by a stone impacted in the duct. The duct from the submandibular gland drain into the floor of the mouth and purulent discharge from this duct causes a foul taste in the mouth.

232
Q

Oesophageal adenocarcinoma is associated with

A

GORD or Barrett’s

233
Q

Constipation in children history

A

poor diet and fluid intake
history of cerebral palsy
learning difficulties, or spinal cord problems;
psychological factors (e.g., depression, abuse, ADHD, autism, oppositional disorder),
weaning, toilet training, start of schooling or other causes of stress may be present; vague abdominal pain, painful defecation (infants may extend their legs and squeeze anal and buttock muscles to prevent stooling;

234
Q

constipation in children exam

A

examination findings may be minimal (mild abdominal tenderness, stool in rectum); abdominal distension in severe cases or in small children; faecal mass palpable on abdominal or rectal examination

235
Q

constipation in children management

A
  1. Dietary changes remain a common initial recommendation, particularly increased intake of fluids and dietary fibre.
  2. osmotic agents ie lactulose or polyethylene glycol (PEG) (quicker response, over 1)
  3. Some children may require the addition of a short-term stimulant laxative (e.g., senna) to achieve regular bowel movements.

If impaction present given osmotic lax 1st line

236
Q

Appendicitis history

A

history of sharp or stabbing periumbilical pain that migrates to the right lower quadrant (RLQ); anorexia, fever, vomiting, and/or diarrhoea may be present; occurs in all age groups but is rare in infants

237
Q

Appendicitis exam

A

positive McBurney’s sign (RLQ pain and tenderness to palpation at a point two-thirds along a line from the umbilicus to the anterior superior iliac spine); positive Rovsing’s sign (pain in the RLQ in response to left-sided palpation, suggesting peritoneal irritation); positive psoas sign (pain in the RLQ when child placed on left side and right hip gently hyperextended, suggesting irritation to the psoas fascia and muscle); positive obturator sign (RLQ pain on internal rotation of the flexed right thigh);

238
Q

Appendicitis management

A

Seek immediate surgical input and consider involving critical care for any patient with suspected appendicitis and signs of shock or sepsis.

Think fluids, abx and access - consider if patient is acutely unwell and in shock.

Laparoscopic appendicectomy is the first choice for most adults

239
Q

Gastroenteritis history

A

vague abdominal pain with nausea and vomiting; diarrhoea with or without mucus in stool; recent travel or contact with sick individual(s) or ingestion of suspected food and drink; >10 days suggests parasitic or non-infectious cause; fever, chills, myalgia, rhinorrhoea, upper respiratory symptoms

240
Q

Gastroenteritis on exam

A

diffuse abdominal pain without evidence of peritonitis (no guarding or rebound tenderness); abdominal distension; hyperactive bowel sounds; mucus in stool (bacterial or parasitic); signs of volume depletion (tachycardia, hypotension, dry mucous membranes, poor capillary refill, sunken fontanelle in infants); low-grade fever, lethargy and/or irritability, reduced response to noxious stimuli, abnormal temperature

241
Q

Gastroenteritis management

A

access dehydration

rehydrate if needed

Daily fluid maintenance requirement is 100 mL/kg for the first 10 kg body weight, 50 mL/kg for the next 10 kg, and 20 mL/kg for each subsequent 1 kg over 20 kg. This is usually supplied by the age-appropriate diet.
For infants who are breast-fed, breastfeeding should be continued.

ondansetron should be considered when vomiting interferes with ORT.

242
Q

UTI in children management

A

age dependant

Neonates under 6 weeks
- IV abx

infants >6 weeks to <2 years children and adolescents : systemically stable - oral cefixime

if unstable - need IV abx

243
Q

Intussuspection presentation / history

A

usually infant between 3 months and 12 months of age presenting with colicky abdominal pain, flexing of the legs, fever, lethargy, and vomiting; Henoch-Schonlein purpura (HSP) may be initiating factor in an older child (usually <11 years of age); vague abdominal complaints; severe, cramp-like abdominal pain; child may be inconsolable

244
Q

Intussuspection exam

A

may see gross or occult blood that may be mixed with mucus and have ‘red currant jelly ‘ appearance, abdominal tenderness, and palpable abdominal mass; signs of HSP may be present in older child (rash of palpable purpura, blood in the stools)

confirm with USS

245
Q

Intussuspection treatment

A

1st line - fluid resuscitation Reduction should be first attempted by contrast enema

2nd (when contrast enema is contra ie in peritonitis, perforation, and hypovolaemic shock) - surgical reduction

246
Q

merkles diverticulum history

A

typically aged <2 years; may present with abdominal pain (may be intermittent or mimic acute appendicitis), and/or painless passage of bright red blood per rectum (haematochezia); often asymptomatic

247
Q

merkles diverticulum exam

A

dark red, maroon, or ‘red currant jelly’ stools; abdominal tenderness with guarding and rebound (may suggest diverticulitis);

248
Q

merkles diverticulum investigation

A

Pertechnetate scan is best

ectopic focus or ‘hot spot’; enhancement of diverticulum

249
Q

merkles diverticulum management

A

symptomatic - surgical excision of the diverticulum

250
Q

crohns disease history

A

crampy abdominal pain, intermittent diarrhoea, bloody diarrhoea if colitis a feature (blood less common in Crohn’s disease than in ulcerative colitis), weight loss, fatigue, family history of inflammatory bowel disease

251
Q

crohns disease exam

A

ulcers, evidence of weight loss, pallor, abdominal tenderness, abdominal mass, perianal fistula, perirectal abscess, anal fissure, perianal skin tags; extraintestinal manifestations including iritis, arthritis, sacroiliitis, erythema nodosum, pyoderma gangrenosum

252
Q

crohns disease investigation

A

FBC:
leukocytosis, anaemia, thrombocytosis

CRP and ESR - raised

colonoscopy with biopsy:
may demonstrate inflammation, friability, ulcer formation, and oedema

253
Q

crohns disease management

A

budesonide - a modified-release corticosteroid, which is active particularly at the terminal ileum

prednisolone: 0.5 to 0.75 mg/kg/day orally, taper gradually according to response (consider adverse effect profile)

immunmodulator therapy ie azathioprine
or mercaptopurine: or methotrexate (with folic acid)

biological therapy

TNF-alpha inhibitors infliximab and adalimumab

254
Q

testicular torsion history

A

acute-onset testicular pain; nausea, and vomiting; history of recurrent episodes suggests repeated episodes of testicular torsion followed by spontaneous detorsion; history of trauma may be present

255
Q

testicular torsion exam

A

tender, oedematous testicle; affected testicle may appear higher than unaffected testicle with horizontal lie; associated scrotal erythema and oedema; absent cremasteric reflex; usually no pain relief with elevation of the scrotum

256
Q

testicular torsion investigations

A

duplex Doppler ultrasound of scrotum:
presence of fluid and the whirlpool sign (the swirling appearance of the spermatic cord from torsion as the ultrasound probe scans downwards perpendicular to the spermatic cord); absent or decreased blood flow in the affected testicle

257
Q

giardiasis history

A

history of travel, contaminated water/food, IgA deficiency, foul-smelling watery/fatty stools, abdominal pain, bloating, or weight loss

258
Q

giardiasis investigation

A

giardiasis - stool culture - This remains the most definitive investigation.

Inexpensive, but requires microscopic expertise and specialised laboratories.

259
Q

giardiasis treatment

A

metronidazole 1st line

Nitroimidazoles (e.g., metronidazole and tinidazole) are first choice for these patients and bind with parasite DNA causing death of the trophozoite.

paromomycin in pregnancy (1st Trimenster`0

260
Q

Labyrinthitis history

A

history of vertigo, dizziness, hearing loss, tinnitus, otalgia, and flu-like symptoms; irritation of the vestibular system can be secondary to trauma, central nervous system or ear infection, or vestibular neuritis

261
Q

Labyrinthitis risk factors

A

Key risk factors include viral infections, acute or chronic otitis media, meningitis, cholesteatoma, and inner ear malformations.

262
Q

Labyrinthitis treatment

A

Symptoms of acute vertigo episodes can be treated with vestibular suppressants and anti-emetics

ie promethazine:

263
Q

malrotation history

A

onset <1 month age with bilious vomiting; more concerning symptoms include haematochezia, abdominal distension, and shock; for older children, presents as chronic vomiting and poor weight gain

264
Q

malrotation exam

A

examination initially normal but may demonstrate rapid progression to acute abdomen secondary to bowel necrosis; there is a high risk of midgut volvulus and intestinal necrosis

265
Q

malroation investigation

A

plain abdominal x-ray:
obstruction: dilatation of the stomach and duodenum

needs emergency surgery

266
Q

aaa screening

A

In England, abdominal aortic aneurysm screening (AAA) is offered to men during the year they turn 65.

If normal (<3cm) the patient will require no further future scans as the chances of developing a AAA after 65 years old is small.

267
Q

Carbimazole

A

hyrotoxicosis. It is typically given in high doses for 6 weeks until the patient becomes euthyroid before being reduced.

Mechanism of action
blocks thyroid peroxidase from coupling and iodinating the tyrosine residues on thyroglobulin → reducing thyroid hormone production
in contrast propylthiouracil as well as this central mechanism of action also has a peripheral action by inhibiting 5’-deiodinase which reduces peripheral conversion of T4 to T3

Adverse effects
agranulocytosis
crosses the placenta, but may be used in low doses during pregnancy

268
Q

Argyll-Robertson pupil

A

Argyll-Robertson pupil is one of the classic pupillary syndrome. It is sometimes seen in neurosyphilis. A mnemonic used for the Argyll-Robertson Pupil (ARP) is Accommodation Reflex Present (ARP) but Pupillary Reflex Absent (PRA)

Features
small, irregular pupils
no response to light but there is a response to accommodate

Causes
diabetes mellitus
syphilis

269
Q

Orthostatic hypotension

A

Orthostatic hypotension can be diagnosed when there is:

a. A drop in systolic BP of 20mmHg or more (with or without symptoms)
b. A drop to below 90mmHg on standing even if the drop is less than 20mmHg (with or without symptoms)
c. A drop in diastolic BP of 10mmHg with symptoms (although clinically much less significant than a drop in systolic BP).

270
Q

SSRI + NSAID = =

A

GI bleeding risk - give a PPI

271
Q

Genital warts

A

90% are caused by HPV 6 & 11

Management
topical podophyllum or cryotherapy are commonly used as first-line treatments depending on the location and type of lesion. Multiple, non-keratinised warts are generally best treated with topical agents whereas solitary, keratinised warts respond better to cryotherapy
imiquimod is a topical cream which is generally used second line
genital warts are often resistant to treatment and recurrence is common although the majority of anogenital infections with HPV clear without intervention within 1-2 years

272
Q

Venous leg ulcers

presentation

A

Most due to venous hypertension, secondary to chronic venous insufficiency (other causes include calf pump dysfunction or neuromuscular disorders)
Ulcers form due to capillary fibrin cuff or leucocyte sequestration

Features of venous insufficiency include oedema, brown pigmentation, lipodermatosclerosis, eczema
Location above the ankle, painless
Deep venous insufficiency is related to previous DVT and superficial venous insufficiency is associated with varicose veins

Doppler ultrasound looks for presence of reflux and duplex ultrasound looks at the anatomy/ flow of the vein
Management: 4 layer compression banding after exclusion of arterial disease or surgery
If fail to heal after 12 weeks or >10cm2 skin grafting may be needed

273
Q

Marjolin’s ulcer presentation

A

Squamous cell carcinoma
Occurring at sites of chronic inflammation e.g; burns, osteomyelitis after 10-20 years
Mainly occur on the lower limb

274
Q

Arterial ulcers presentation

A
Occur on the toes and heel
Painful
There may be areas of gangrene
Cold with no palpable pulses
Low ABPI measurements
275
Q

Neuropathic ulcers presentation

A

Commonly over plantar surface of metatarsal head and plantar surface of hallux
The plantar neuropathic ulcer is the condition that most commonly leads to amputation in diabetic patients
Due to pressure
Management includes cushioned shoes to reduce callous formation

276
Q

Pyoderma gangrenosum presentation

A

Associated with inflammatory bowel disease/RA
Can occur at stoma sites
Erythematous nodules or pustules which ulcerate

277
Q

retractile testis

A

testis that appears in warm conditions or which can be brought down on clinical examination and does not immediately retract

278
Q

neoplastic spinal cord compression is suspected

A

high-dose oral dexamethasone should be given whilst awaiting investigations

279
Q

benign prostatic hyperplasia management

A

Alpha-1 antagonists

280
Q

BPH typically presents with lower urinary tract symptoms (LUTS), which may be categorised into:

A
voiding symptoms (obstructive): weak or intermittent urinary flow, straining, hesitancy, terminal dribbling and incomplete emptying
storage symptoms (irritative) urgency, frequency, urgency incontinence and nocturia
post-micturition: dribbling
complications: urinary tract infection, retention, obstructive uropathy
281
Q

prophylaxis for contacts of patients with meningococcal meningitis

A
Oral ciprofloxacin (preferred ) 
( or rifampicin)
282
Q

most useful prognostic marker in paracetamol overdose?

A

An elevated prothrombin time signifies liver failure in paracetamol overdose and is a marker of poor prognosis.

283
Q

acute pericarditis management

A
  1. combination of NSAID and colchicine
284
Q

Otosclerosis describes the replacement of normal bone by vascular spongy bone. It causes a progressive conductive deafness due to fixation of the stapes at the oval window. Otosclerosis is autosomal dominant and typically affects young adults

features

A
Onset is usually at 20-40 years - features include:
conductive deafness
tinnitus
normal tympanic membrane*
positive family history
285
Q

Otosclerosis management

A

hearing aid

286
Q

bstructive urinary calculi and signs of infection

A

urgent renal decompression and IV antibiotics due to the risk of sepsis

287
Q

Sarcoidosis is a multisystem disorder of unknown aetiology characterised by non-caseating granulomas. It is more common in young adults and in people of African descent

Features

A

acute: erythema nodosum, bilateral hilar lymphadenopathy, swinging fever, polyarthralgia
insidious: dyspnoea, non-productive cough, malaise, weight loss
skin: lupus pernio
hypercalcaemia: macrophages inside the granulomas cause an increased conversion of vitamin D to its active form (1,25-dihydroxycholecalciferol)

288
Q

Lyme disease management

A

Management of suspected/confirmed Lyme disease
doxycycline if early disease. Amoxicillin is an alternative if doxycycline is contraindicated (e.g. pregnancy)
people with erythema migrans should be commenced on antibiotic without the need for further tests
ceftriaxone if disseminated disease
Jarisch-Herxheimer reaction is sometimes seen after initiating therapy: fever, rash, tachycardia after first dose of antibiotic (more commonly seen in syphilis, another spirochaetal disease)

289
Q

OCD management

A

he most appropriate initial management would be a form of low-intensity psychological therapy. NICE guidelines recommend exposure and response prevention as a beneficial therapy for patients with OCD.

The use of selective serotonin re-uptake inhibitors is also recommended by NICE guidelines, but is typically commenced following a stepwise approach where non-pharmacological interventions are recommended first.

290
Q

Chronic insomnia

A

may be diagnosed after three months, if a person has trouble falling asleep or staying asleep at least three nights per week

291
Q

medication in insomnia

A

hypnotics:
The hypnotics recommended for treating insomnia are short-acting benzodiazepines or non-benzodiazepines (zopiclone, zolpidem and zaleplon).
Diazepam is not recommended but can be useful if the insomnia is linked to daytime anxiety.
Use the lowest effective dose for the shortest period possible.
If there has been no response to the first hypnotic, do not prescribe another. You should make the patient aware that repeat prescriptions are not usually given.
It is important to review after 2 weeks and consider referral for cognitive behavioural therapy (CBT).

292
Q

oculogyric crisis

A

oculogyric” refers to the bilateral elevation of the visual gaze, but several other responses are associated with the crisis.
= dsytonia

293
Q

Tricyclic antidepressants (TCAs) ie imipramine side effects

A
Common side-effects
drowsiness
dry mouth
blurred vision
constipation
urinary retention
lengthening of QT interval

antimuscarinic side-effects are more common with imipramine than other types of tricyclic antidepressants.

294
Q

Akathisia

A

sense of inner restlessness and inability to keep still

295
Q

Metformin should be stopped f when

A

following a myocardial infarction due to the risk of lactic acidosis.

296
Q

typical carboxyhaemoglobin levels

A

3% non-smokers
< 10% smokers
10 - 30% symptomatic: headache, vomiting
> 30% severe toxicity

297
Q

Drug-induced thrombocytopenia (probable immune-mediated)

A

quinine
abciximab
NSAIDs
diuretics: furosemide
antibiotics: penicillins, sulphonamides, rifampicin
anticonvulsants: carbamazepine, valproate
heparin

298
Q

Ethylene glycol toxicity management (antifreeze)

A

Fomepizole (IV), an inhibitor of alcohol dehydrogenase, is used as the first-line treatment for ethylene glycol toxicity. Fomepizole is superior antidote than ethanol because it is safer and easy to dose and administer.

299
Q

Contraindications of metformin

A

chronic kidney disease: NICE recommend that the dose should be reviewed if the creatinine is > 130 µmol/l (or eGFR < 45 ml/min) and stopped if the creatinine is > 150 µmol/l (or eGFR < 30 ml/min)
metformin may cause lactic acidosis if taken during a period where there is tissue hypoxia. Examples include a recent myocardial infarction, sepsis, acute kidney injury and severe dehydration
iodine-containing x-ray contrast media: examples include peripheral arterial angiography, coronary angiography, intravenous pyelography (IVP); there is an increasing risk of provoking renal impairment due to contrast nephropathy; metformin should be discontinued on the day of the procedure and for 48 hours thereafter
alcohol abuse is a relative contraindication

300
Q

Drugs to avoid in renal failure

A

antibiotics: tetracycline, nitrofurantoin
NSAIDs
lithium
metformin

301
Q

Drugs likely to accumulate in chronic kidney disease - need dose adjustment in renal failure

A
most antibiotics including penicillins, cephalosporins, vancomycin, gentamicin, streptomycin
digoxin, atenolol
methotrexate
sulphonylureas
furosemide
opioids
302
Q

Rifampicin tb

A

mechanism of action: inhibits bacterial DNA dependent RNA polymerase preventing transcription of DNA into mRNA
potent liver enzyme inducer
hepatitis, orange secretions
flu-like symptoms

303
Q

Isoniazid tb

A

mechanism of action: inhibits mycolic acid synthesis
peripheral neuropathy: prevent with pyridoxine (Vitamin B6)
hepatitis, agranulocytosis
liver enzyme inhibitor

304
Q

Pyrazinamide

tb

A

mechanism of action: converted by pyrazinamidase into pyrazinoic acid which in turn inhibits fatty acid synthase (FAS) I
hyperuricaemia causing gout
arthralgia, myalgia
hepatitis

305
Q

Ethambutol tb

A

mechanism of action: inhibits the enzyme arabinosyl transferase which polymerizes arabinose into arabinan
optic neuritis: check visual acuity before and during treatment
dose needs adjusting in patients with renal impairment

306
Q

Methotrexate monitoring

A

FBC, LFT, U&E

307
Q

Caution should always be exercised when combining diuretics. However, which one of the following combinations is always contraindicated?

A

Amiloride and spironolactone a

hyperkalemia

308
Q

Steroid responsiveness COPD

A

Asthmatic features/features suggesting steroid responsiveness in COPD:
previous diagnosis of asthma or atopy
a higher blood eosinophil count
substantial variation in FEV1 over time (at least 400 ml)
substantial diurnal variation in peak expiratory flow (at least 20%)

309
Q

Acromegaly: management

A

Trans-sphenoidal surgery is the first-line treatment for acromegaly in the majority of patients

Dopamine agonists
for example bromocriptine
the first effective medical treatment for acromegaly, however now superseded by somatostatin analogues
effective only in a minority of patients

Somatostatin analogue
directly inhibits the release of growth hormone
for example octreotide
effective in 50-70% of patients
may be used as an adjunct to surgery

Pegvisomant
GH receptor antagonist - prevents dimerization of the GH receptor
once daily s/c administration
very effective - decreases IGF-1 levels in 90% of patients to normal
doesn’t reduce tumour volume therefore surgery still needed if mass effect

310
Q

Drug-induced urinary retention

A

The following drugs may cause urinary retention:
tricyclic antidepressants e.g. amitriptyline
anticholinergics
opioids
NSAIDs
disopyramide

311
Q

Thyrotoxic storm treatment

A

beta blockers, propylthiouracil and hydrocortisone

312
Q

ACR results

A

NICE recommendations for referral to a nephrologist:
a urinary albumin:creatinine ratio (ACR) of 70 mg/mmol or more, unless known to be caused by diabetes and already appropriately treated
a urinary ACR of 30 mg/mmol or more, together with persistent haematuria (two out of three dipstick tests show 1+ or more of blood) after exclusion of a urinary tract infection
consider referral to a nephrologist for people with an ACR between 3-29 mg/mmol who have persistent haematuria and other risk factors such as a declining eGFR, or cardiovascular disease

Management
ACE inhibitors (or angiotensin II receptor blockers) are key in the management of proteinuria
they should be used first-line in patients with coexistent hypertension and CKD
if the ACR > 70 mg/mmol they are indicated regardless of the patient’s blood pressure

313
Q

open angle glaucoma treatment

A

NICE guidelines:
first line: prostaglandin analogue (PGA) eyedrop
second line: beta-blocker, carbonic anhydrase inhibitor, or sympathomimetic eyedrop
if more advanced: surgery or laser treatment can be tried2

314
Q

open angle glaucoma features

A

Features
severe pain: may be ocular or headache
decreased visual acuity
symptoms worse with mydriasis (e.g. watching TV in a dark room)
hard, red-eye
haloes around lights
semi-dilated non-reacting pupil
corneal oedema results in dull or hazy cornea
systemic upset may be seen, such as nausea and vomiting and even abdominal pain

315
Q

Metformin predisposes to lactic acidosis

A

should therefore be used with caution in patients with acute kidney injury

316
Q

adverse effects if metformin

A

dverse effects
gastrointestinal upsets are common (nausea, anorexia, diarrhoea), intolerable in 20%
reduced vitamin B12 absorption - rarely a clinical problem
lactic acidosis* with severe liver disease or renal failure

317
Q

Nerve at risk during a total hip replacement

A

Sciatic nerve

318
Q

MRSA management

A

uppression of MRSA from a carrier once identified

nose: mupirocin 2% in white soft paraffin, tds for 5 days
skin: chlorhexidine gluconate, od for 5 days. Apply all over but particularly to the axilla, groin and perineum

The following antibiotics are commonly used in the treatment of MRSA infections:
vancomycin
teicoplanin
linezolid

319
Q

Headache: red flags

A

compromised immunity, caused, for example, by HIV or immunosuppressive drugs
age under 20 years and a history of malignancy
a history of malignancy known to metastasis to the brain
vomiting without other obvious cause
worsening headache with fever
sudden-onset headache reaching maximum intensity within 5 minutes - ‘thunderclap’
new-onset neurological deficit
new-onset cognitive dysfunction
change in personality
impaired level of consciousness
recent (typically within the past 3 months) head trauma
headache triggered by cough, valsalva (trying to breathe out with nose and mouth blocked), sneeze or exercise
orthostatic headache (headache that changes with posture)
symptoms suggestive of giant cell arteritis or acute narrow-angle glaucoma
a substantial change in the characteristics of their headache

320
Q

mild to moderate dementia management

A

Acetylcholinesterase (AChE) inhibitors (donepezil, galantamine, and rivastigmine) [NICE, 2006; NICE, 2011]:
These drugs can be used for mild to moderate Alzheimer’s disease but must only be prescribed by healthcare professionals with expertise in this area …

Memantine is an option for managing Alzheimer’s disease for people with:
Moderate Alzheimer’s disease who are intolerant of, or have a contraindication to, AChE inhibitors or
Severe Alzheimer’s disease.

321
Q

tonsilitis criteria

A
The criteria are as follows:
• Temperature >38ºC
• Tender anterior cervical lymphadenopathy
• Absence of a cough
• Exudate present on tonsils
322
Q

TURP syndrome what is this

A

TURP syndrome is a rare and life-threatening complication of transurethral resection of the prostate surgery.

It is caused by irrigation with large volumes of glycine, which is hypo-osmolar and is systemically absorbed when prostatic venous sinuses are opened up during prostate resection. This results in hyponatremia, and when glycine is broken down by the liver into ammonia, hyper-ammonia and visual disturbances.

323
Q

RF TURP syndrome

A
There are risk factors for developing TURP syndrome are : 
surgical time > 1 hr
height of bag > 70cm
resected > 60g
large blood loss
perforation
large amount of fluid used
poorly controlled CHF
324
Q

Adhesive capsulitis presents

A

painful stiff shoulder with restriction of active and passive range of motion in abduction, internal and external rotation.

325
Q

Sensineural hearing loss

A

t has a diverse range of pathologies including common causes such as:

Presbycusis (most common)
Noise-induced hearing loss
Congenital infections (e.g. rubella, CMV)
Neonatal complications (e.g. kernicterus or meningitis)
Drug induced deafness (aminoglycosides).
Vascular pathology (stroke, transient ischaemic attacks)

326
Q

Pharmacodynamic interactions with Methotrexate

A

Trimethoprim and Methotrexate are anti-folate agents. This means that there is an increased risk of Methotrexate toxicity,

327
Q

AKI following contrast risks

A

An increased risk of AKI is associated with: chronic kidney disease, diabetes with chronic kidney disease, heart failure, renal transplant, age 75 or over, hypovolaemia, increasing volume of contrast agent, and intra-arterial administration of contrast agent.

328
Q

Pharmacological management of chronic COPD

A

Step 1
Starting with a Short acting B2 agonist (SABA)/ short acting muscarinic antagonist (SAMA). These are continued as the patient goes up the management steps.

Step 2
For patients with persistent exacerbations but no asthmatic features (e.g. asthma/atopy history, raised eosinophils, reduced FEV1 or diurnal variations in peak flow) or evidence of steroid responsiveness then add a long acting B2 agonist (LABA) AND a long acting muscarinic antagonist (LAMA).

For patients with persistent exacerbations with asthmatic features or evidence of steroid responsiveness increase management to long acting B2 agonist (LABA) and inhaled corticosteroid in combination (ICS).

Step 3
If patients on a LABA + LAMA combination are still getting daily symptoms that affect their activities of daily living then a 3 month trial of LAMA + LABA + ICS should be considered. If this does not work then it should be reverted back to LABA + LAMA.

If any patient on Step 2 is getting more than one severe or two moderate exacerbations in a year then LAMA + LABA + ICS should be started.

Step 4
If patients are still symptomatic consider specialist referral.

329
Q

drug induced lupus

A

Drug induced lupus (DIL) typically presents as a less severe form of systemic lupus erythematosus (SLE) which starts while taking an offending medication, and completely resolves after it is discontinued. Although DIL can present with any feature of SLE, the most common features are:
Systemic upset (fever / fatigue / myalgia).
Arthritis (non-erosive) or arthralgia.
Serositis in the form of pleurisy or pericarditis.

check for

330
Q

Acquired melanocytic naevus

A

Acquired melanocytic naevus
Occur in childhood and evolve through a number of steps:

Junctional: flat + pigmented (due to melanocytes at the basal layer of the epidermis)
Compound: raised + pigmented (due to melanocytes at the basal layer of the epidermis + deep in the dermis)
Intradermal: raised + pale (due to melanocytes deep in the dermis)

331
Q

Antibodies associated with dermatomyositis

A

constitutional symptoms, symmetrical muscle weakness and a characteristic rash (heliotrope rash, shawl rash, mechanic’s hands). Creatine kinase is markedly raised and Anti-Jo1 antibodies are characteristically raised as well. A muscle biopsy will show evidence of Dermatomyositis.

332
Q

Imatinib

A

Imatinib is a tyrosine kinase inhibitor

treatment of chronic myeloid leukaemia (CML) in 2001.

Common side effects include fluid retention, headache, diarrhoea, loss of appetite, weakness, nausea and vomiting, abdominal distension, oedema, rash, dizziness, and muscle cramps. Serious side effects may include myelosuppression, heart failure, and liver function abnormalities.

333
Q

Causes of high output cardiac failure

A

auses of high output cardiac failure can be remembered by the mnemonic AAPPTT:

Anaemia
Arteriovenous malformation
Paget's disease
Pregnancy
Thyrotoxicosis
Thiamine deficiency (wet Beri-Beri)
334
Q

Complications of nephrotic syndrome

A

lose immunoglobulins in their urine. This is because the damage to the glomerular capillary walls causes increased permeability to proteins. This means that these patients are at an increased susceptibility to infection. In particular they are susceptible to staphylococcal and pneumococcal infections.

335
Q

Peri-operative Management of Insulin Dependent Diabetes

A

Peri-operative management principles of insulin use are:

Put the patient as early on the theatre list as possible minimising the amount of time the patient is nil by mouth.
Stop long acting insulin the night before.
Stop any other insulin and begin sliding scale insulin infusion from when the patient is placed nil by mouth.
Continue infusion until patient is able to eat post-operatively.
Switch to normal insulin regimen around their first meal.

336
Q

broad complex tachycardia

A

A broad complex tachycardia will have a heart rate of more than 100 and widened QRS complexes. These patients can be stable or unstable.

Patients with a broad complex tachycardia should be managed according to the ALS algorithm. All patients should be maintained on high flow oxygen and IV access obtained.

Adverse signs are:

Blood pressure <90mmHg (systolic)
Heart rate >150
Chest pain
Heart failure
If a patient displays adverse signs, the patient should be prepared for DC cardioversion.

If the patient is stable, they should be started on 300mg IV amiodarone over 1h followed by an infusion of 900mg amiodarone over 24h.

The patient should be urgently reviewed by cardiology and intensive care.

337
Q

Haemofiltration should be considered if the following are present;

A
Pulmonary oedema
Refractory hyperkalaemia
Severe metabolic acidosis
Uraemia
Drug overdose e.g. aspirin overdose
338
Q

Muscle spasm pain in palliative care

A

The management of pain in palliative care patients in important but can be challenging, and a history is key to ascertaining the best course of treatment.

Simple, conservative measures such as heat, massage and relaxation should always be considered.

Transcutaneous electric nerve stimulation can be used if there is a specific trigger point.

PO diazepam is the first line treatment for spasmodic pain, titrated to effect. Baclofen is a GABA receptor agonist may also be used as a second line.

339
Q

Torsades de points

A

orsades de points is a polymorphic ventricular tachycardia, a heart rhythm that may arise in a patient with a prolonged QT interval.

Patients present with feeling generally unwell, palpitations, chest pain, dizziness and syncope. Sudden cardiac death may also occur.

Causes of torsades de points include;

Abnormal electrolytes
Drugs - clarithromycin, anti-psychotics, fluoxetine, cimetidine, lithium, sotalol
Genetic conditions
Treatment of torsades de points is primarily supportive using the ABCDE algorithm. DC cardioversion is indicated if the patient is unstable. Isoprenaline and pacing can be used if the patient is stable, and advice should be sought from a specialist.

340
Q

chronic mesenteric ischaemia

A

Chronic mesenteric ischaemia typically presents in elderly patients. Patients have risk factors for arterial disease (smoking, diabetes, high cholesterol, previous MI etc.) or embolic disease (such as atrial fibrillation).

Patients typically present with diffuse, colicky abdominal pain which is worse after eating. Weight loss is a common feature, as patients avoid eating due to the pain. There may be associated diarrhoea and melaena or haematochezia (secondary to mucosal sloughing).

On physical examination there is commonly abdominal tenderness. An epigastric bruits may be present (secondary to turbulent flow in the narrowed vessels).

341
Q

Management of narrow complex tachycardias (regular)

A
ABCDE 
adverse features (shock, syncope, heart failure, or myocardial ischaemia), emergency synchronised direct current (DC) cardioversion is indicated

trial vagal manoeuvres (carotid sinus massage or Valsalva manoeuvre).
If vagal manoeuvres fail, adenosine should be administered (initially as a 6 mg intravenous bolus, and if this fails 12 mg followed by a further 12 mg is trialled).

342
Q

Management of narrow complex tachycardias (irregular)

A
ABCDE 
adverse features (shock, syncope, heart failure, or myocardial ischaemia), emergency synchronised direct current (DC) cardioversion is indicated

Atrial fibrillation with onset less than 48 hours is typically managed with rhythm control (LMWH followed by flecainide if there is no structural heart disease, or amiodarone if there is structural heart disease).

Atrial fibrillation with onset more than 48 hours is typically managed with rate control (i.e. metoprolol or bisoprolol or verapamil, or digoxin if there are signs of heart failure) and anticoagulation

343
Q

investigation for aortic dissection diagnosis

A

CT angiogram is used to diagnose dissection

344
Q

Acute causes of AR

A

Infective endocarditis, with valve destruction and leaflet perforation. It can also cause perivalvular abscesses, which can rupture into the left ventricle
Aortic dissection, which can cause regurgitation by primarily impeding valve closure
Traumatic rupture of the valve leaflets, caused by blunt chest trauma or deceleration injury
Iatrogenic causes include balloon valvotomy or trans catheter aortic valve implantation (TAVI)

345
Q

Chronic causes of AR

A

Chronic valvular causes of AR include:

Calcific aortic valve disease (age related)
Myxomatous degeneration
Congenital disease e.g. bicuspid aortic valve
Rheumatic heart disease - most common cause in the developing world
Infective endocarditis
Rheumatic causes e.g. rheumatoid arthritis, antiphospholipid syndrome
Marfan’s syndrome

346
Q

Presentation of Acute AR

A
Sudden cardiovascular collapse
Pulmonary oedema
Pallor
Sweating
Peripheral vasoconstriction

(Early diastolic murmur)

347
Q

Presentation of Chronic AR

A

Exertional dyspnoea
Orthopnoea
Paroxysmal nocturnal dyspnoea
Stable angina can also develop in some patients with severe AR, even in the absence of coronary artery disease. This is due to reduction in diastolic coronary perfusion.

(Early diastolic murmur)

348
Q

Investigations for AR

A

Echocardiogram is the definitive investigation for aortic regurgitation (AR).

349
Q

Surgical intervention in aortic regurgitation (AR) is indicated in the following situations

A

Significant enlargement of the ascending aorta
Symptomatic AR
Asymptomatic AR with the following parameter findings:
Poor left ventricular ejection fraction (<= 50%)
Left ventricular end diastolic diameter more than 70mm or left ventricular end systolic diameter mote than 50mm
Infective endocarditis refractory to medical therapy

350
Q

Aortic stenosis (AS) is associated with a classic triad of symptoms:

A

Heart failure
Syncope
Angina

351
Q

ECG findings for AS

A

Increased QRS complex voltage
Left axis deviation
Poor R-wave progression

352
Q

AS can be judged to be severe via the following parameters

A

Peak gradient > 40 mmHg (note, in severe left ventricular dysfunction, a low peak gradient can be falsely reassuring)

Valve area < 1.0 cm^2

Aortic jet velocity >4 m/s

353
Q

ndications for intervention in AS

A

All patients with symptomatic aortic stenosis
Asymptomatic patients with a left ventricular ejection fraction (LVEF) < 50%
Asymptomatic patients with an LVEF > 50% who are physically active, and who have symptoms or a fall in blood pressure during exercise testing

Aortic valve peak velocity > 5.5 m/s^2
Severe calcification and peak velocity progression >= 0.3 m/s^2
Markedly elevated BNP levels without other explanation
Severe pulmonary hypertension (pulmonary artery systolic pressure > 60mmHg)

354
Q

intervention in AS

A

TAVI is favoured with patients with severe comorbidities, previous heart surgery, frailty, restricted mobility, and those older than 75 years of age.

SAVR is favoured for patients who are low risk and less than 75 years of age.

355
Q

CHADS2VASc score components

A

C: 1 point for congestive cardiac failure.
H: 1 point for hypertension.
A2: 2 points if the patient is aged 75 or over.
D: 1 point if the patient has diabetes mellitus.
S2: 2 points if the patient has previously had a stroke or transient ischaemic attack (TIA).
V: 1 point if the patient has known vascular disease.
A: 1 point if the patient is aged 65-74.
Sc: 1 point if the patient is female.

356
Q

Management of VF

A

Management is according to the Advanced Life Support guidelines:

The initial priorities will be as for the Basic Life Support: ensure the airway is patent, check for signs of life (pulse and breathing), and commence CPR.

Ventricular fibrillation is a shockable rhythm: the next step is to administer defibrillation (unsynchronised cardioversion using a 200 J biphasic shock).

Chest compressions should then be resumed.

1 mg adrenaline (10 ml 1:10 000) plus 300 mg amiodarone should be administered after the 3rd shock. Adrenaline should subsequently be administered every 3-5 mins (after every alternate shock).

357
Q

Management of pulseless Ventricular tachycardia (VT)

A

If there is no pulse the patient should be managed according to the Advanced Life Support algorithm:

VT is a shockable rhythm so a 200 J bi-phasic (unsynchronised) shock should be administered.
CPR should be resumed for 2 minutes before re-checking the rhythm.
Intravenous adrenaline (1 mg of 10 ml 1:10 000 solution) and amiodarone (300 mg) should be administered after delivery of the 3rd shock.
Adrenaline should be administered every 3-5 minutes thereafter (after every alternate shock).

358
Q

Management of Ventricular tachycardia with a pulse with adverse features

A

If there is a pulse but the patient shows adverse features (shock, syncope, myocardial ischaemia, or heart failure) the patient should be managed according to the Resuscitation Council tachyarrhythmia algorithm:

Administer a synchronised DC shock (up to 3 attempts).
After seeking expert help amiodarone (300 mg intravenously over 10-20 minutes followed by 900 mg over 24 hours) should be administered.

359
Q

Management of Ventricular tachycardia with a pulse with no adverse feature

A

Management of Ventricular tachycardia with a pulse with no adverse features
Management is with amiodarone (300 mg intravenously over 20-60 minutes followed by 900 mg over 24 hours)

360
Q

Signs of Cardiac Tamponade can be remembered by Beck’s Triad.

A

Signs of Cardiac Tamponade can be remembered by Beck’s Triad.

Hypotension
Quiet heart sounds
Raised JVP

361
Q

Bradyarrhythmias requiring pacemakers

A

Complete heart block (whether asymptomatic or symptomatic)
Mobitz type 2 heart block (whether asymptomatic or symptomatic)
Symptomatic sick sinus syndrome
Permanent bradyarrhythmias caused by a myocardial infarct (typically anterior infarcts - arrhythmias caused by inferior infarcts tend to be temporary)

362
Q

Tachyarrhythmias requiring pacemakers

A

Tachyarrhythmias requiring permanent pacing are those that are resistant to pharmacological therapy.
These forms of permanent pacing are typically dual chamber (with one lead in the right atrium and one lead in the right ventricle).
Bi-ventricular pacemaker (in which a 3rd wire is placed in a branch of the coronary sinus, to allow left ventricular pacing and ventricular synchronisation) is an option if the left ventricular ejection fraction is <35%.

363
Q

Hypertrophic cardiomyopathy

A

Hypertrophic cardiomyopathy is the commonest genetic heart condition. It is often the cause of sudden cardiac death in young people and athletes. (AD)

Many patient are asymptomatic
Generally presents in young adults
Exertional syncope
Shortness of breath
Chest pain
Palpitations
Heart failure
Double apex pulse
Ejection systolic murmur at the lower left sternal edge that may get louder during exercise and reduce if lying supine.
364
Q

ECG findings in MR

A

P-mitrale (a broad notched P wave due to left atrial enlargement)
Right ventricular hypertrophy
Right axis deviation.m

365
Q

Causes of right heart strain

A

Right heart strain is caused by pathologies affecting the pulmonary vasculature such as

Pulmonary emboli
Pulmonary hypertension
Chronic lung disease
Pulmonary stenosis
Pneumothorax
366
Q

ECG features of right heart strain

A
Sinus tachycardia (most common)
Right ventricular strain pattern (ST depression and T wave inversion in right ventricle and inferior leads)
RBBB
Right axis deviation
P pulmonale
S1Q3T3
Atrial arrhythmias
367
Q

Managment of AF

A

1.Aspirin
Statin
Sublingual GTN
Beta blocker or rate limiting calcium channel blocker

  1. beta blocker and long-acting dihydropyridine calcium channel blocker.
  2. A 3rd medication should only be added if the patient is symptomatic despite 2 anti-anginal drugs. Coronary angiography should be arranged unless contraindicated as PCI may be required.
368
Q

Contraindications in WPW

A

Digoxin and NDP-CCBs (e.g. verapamil) are contraindicated for long term use because they may precipitate ventricular fibrillation.

If the patient is experiencing supraventricular tachycardia the management depends on whether the patient is stable or unstable, and if stable the type of arrhythmia:

369
Q

prevent pathological fractures in bone metastases

A

Bisphosphonates and denosumab can be used to prevent pathological fractures in bone metastases. If the eGFR < 30, denosumab is preferred

370
Q

Medical management of a miscarriage

A

vaginal misoprostol alone

371
Q

drugs should be avoided in breast feeding

A
antibiotics: ciprofloxacin, tetracycline, chloramphenicol, sulphonamides
psychiatric drugs: lithium, benzodiazepines
aspirin
carbimazole
methotrexate
sulfonylureas
cytotoxic drugs
amiodarone
372
Q

loop diuretic. where do they act

A

inhibits the Na-K-Cl cotransporter in the thick ascending limb of the loop of Henle

373
Q

Acute angle closure glaucoma and priamry assocaitions

A

Acute angle closure glaucoma

is associated with hypermetropia, where as primary open-angle glaucoma is associated with myopia

374
Q

Water deprivation test: cranial DI

A

urine osmolality after fluid deprivation: low

urine osmolality after desmopressin: high

375
Q

Nephrogenic DI water dep

A

urine osmolality after fluid deprivation: low

urine osmolality after desmopressin: low

376
Q

Symptoms of Hypoalbuminaemia

A
Peripheral oedema (due to reduced oncotic pressure, fluid moves to tissues) - starts in ankles and progresses up the body
Ascites
377
Q

certain circumstances when the creatine equation may not be accurate. Examples include:

A

Amputation - reduced muscle mass and body weight
Pregnancy
Very high or low BMI
High protein meals
Under 18s - never validated
Extremely old patients - never validated and likely to have low muscle mass

378
Q

Hypercalcaemia is defined as correct calcium >2.65mmol/L.

Management

A

Aggressive IV fluids (corrects dehydration, protects the kidneys and increases calcium excretion)
Bisphosphonates (inhibits osteoclast activity reducing calcium release)
Further management to prevent recurrence (depending on the cause):
Chemotherapy (malignancy)
Surgical resection (malignancy)
Radiotherapy (malignancy)
Steroids (sarcoidosis)
Calcitonin
Furosemide

379
Q

Hypernatraemia is defined as serum sodium concentration >145mmol/L.

Clinical Features

A

Lethargy
Weakness
Signs;

Confusion
Agitation
Seizures
Coma

treat with IV fluids

380
Q

Hyperphosphataemia may be treated with

A

phsophate binders (eg. sevelamer 800mg TDS PO during meals)

381
Q

Clinical features of hypocalcaemia can be remembered by the mnemonic ‘SPASMODIC’.

A

SPASMODIC:

S – Spasms (Trousseau's sign)
P – Perioral parasthaesia
A – Anxiety/Irritability
S – Seizures
M – Muscle tone increase (colic, dysphagia)
O– Orientation impairment (i.e. confusion)
D – Dermatitis
I – Impetigo herpetiformis
C – Chvostek's sign
382
Q

Management of severe hypokalaemia

A

Management of severe hypokalaemia
Severe:

Continuous cardiac monitoring
Check and correct magnesium (low magnesium causes renal potassium wasting)
IV infusion of 1L 0.9% saline containing 40mmol potassium chloride.
Note that maximum peripheral potassium infusion rate is 10mmol/hour.
If faster rates required, a central line will neeed to be inserted
Avoid glucose and bicarbonate solutions
Treat cause(s)

383
Q

The management of hyponatraemia depends on the cause:

A
Hypovolaemic:
IV normal saline
Treat underlying cause
Euvoleamic:
SIADH
Fluid restriction
ADH receptor antagonists (e.g. tolvaptan, deomeclocycline)
Oral sodium and furosemide
Hypothyroidism
Levothyroxine
Hypervolaemic:
Fluid restriction
Treat underlying cause
384
Q

Causes of lactic acidosis

A
Tissue hypoxia (Type A)
Shock (e.g. cardiogenic, hypovolaemic, haemorrhagic)
Hypoxia
Acute mesenteric ischaemia
Limb ischaemia
Severe anaemia
Seizures
Vigorous exercise
Abnormalities in metabolism of lactate (Type B)
Diabetic ketoacidosis
Cancer
Liver disease
Inborn errors in metabolism
Drugs:
Metformin - impairs liver metabolism of lactate
Aspirin
385
Q

Normal anion gap acidosis is the result of one of three pathologies:

A

Increased loss of HCO3-, such as;
Diarrhoea
High output ileostomy
Ureterosigmoidostomy

Impaired kidney resorption (e.g. acetazolamide treatment)
Renal tubular acidosis type 2
Hypoaldosteronism
Adrenal insufficiency
Mineralocorticoid receptor block (e.g. spironolactone treatment)

Increased production of H+, such as;
Toluene poisoning
Lysine or arginine administration (found in total parental nutrition)
Decreased excretion of H+, such as;
Renal tubular acidosis types 1 and 4
386
Q

Management of osteomalcia

A

A vitamin D level of < 25 nmol/L = deficiency, requires high dose treatment initially followed by maintenance treatment.

A vitamin D level of between 25-50 nmol/L = insufficiency, can be treated with long term maintenance therapy alone.

Lifestyle advice:

Safe sun exposure
Dietary intake of vitamin D and calcium
Long term maintenance supplements

387
Q

SIADH managment

A

Management revolves around offloading this excess water:

Fluid restriction (up to 750ml/day) and treat underlying cause
ADH antagonists (e.g. tolvaptan, deomeclocycline)
Oral sodium and furosemide
388
Q

vestibular schwannomas which cranial nerves effected

A

Cranial nerves V, VII and VIII

389
Q

cardiogenic shock

PAP CO and SVR

A

Pulmonary artery occlusion pressure - high
CO - Low
Systemic vascular resistance - high

390
Q

Hypovolaemia

PAP CO and SVR

A

Pulmonary artery occlusion pressure - low
CO - Low (decreased preload)
Systemic vascular resistance - high

391
Q

Septic shock

PAP CO and SVR

A

Pulmonary artery occlusion pressure - low
CO - high
Systemic vascular resistance - low

392
Q

prophylaxis of oesophageal bleeding

A

A non-cardioselective B-blocker (NSBB) is used for the prophylaxis of oesophageal bleeding

ie Propranolol

393
Q

e investigation of choice for reflux nephropathy

A

Micturating cystography

394
Q

safer opioid to use in patients with moderate to end-stage renal failure

A

Oxycodone

395
Q

Klinefelter’s syndrome is associated with

A

karyotype 47, XXY.

Features
often taller than average
lack of secondary sexual characteristics
small, firm testes
infertile
gynaecomastia - increased incidence of breast cancer
elevated gonadotrophin levels but low testosterone

Diagnosis is by karyotype (chromosomal analysis).

396
Q

Oesophageal cancer

A

Dysphagia may be associated with weight loss, anorexia or vomiting during eating
Past history may include Barrett’s oesophagus, GORD, excessive smoking or alcohol use

397
Q

Hypertrophic obstructive cardiomyopathy (HOCM)

Management

A
Management
Amiodarone
Beta-blockers or verapamil for symptoms
Cardioverter defibrillator
Dual chamber pacemaker
Endocarditis prophylaxis*
398
Q

Drugs to avoid in HOCM

A

Drugs to avoid
nitrates
ACE-inhibitors
inotropes

399
Q

The British Thoracic Society (BTS) state that the following are the main indications for placing a chest tube in pleural infection:

A

Patients with frankly purulent or turbid/cloudy pleural fluid on sampling should receive prompt pleural space chest tube drainage.

The presence of organisms identified by Gram stain and/or culture from a non-purulent pleural fluid sample indicates that pleural infection is established and should lead to prompt chest tube drainage.

Pleural fluid pH < 7.2 in patients with suspected pleural infection indicates a need for chest tube drainage.

400
Q

confirm stone diagnosis imagine (renal colic)

A

non contrast CT

401
Q

pontaneous bacterial peritonitis: most common organism

A

E. coli

402
Q

HRT for women at risk of venous thromboembolism

A

transdermal

403
Q

Bile-acid malabsorption may be treated with c

A

cholestyramine

404
Q

Hyperemesis gravidarum, diagnostic criteria triad:

A

5% pre-pregnancy weight loss
dehydration
electrolyte imbalance

405
Q

Left ventricular hypertrophy and deep ST depression and T-wave inversions

A

hypertrophic obstructive cardiomyopathy.

406
Q

Charcot’s cholangitis triad:

A

fever, jaundice and right upper quadrant pain

407
Q

Beta-blockers are contraindicated in patients with asthma when managing atrial fibrillation
what do you give instead?

A

CCB ie n. Diltiazem

408
Q

Blepharitis may be associated with

A

seborrhoeic dermatitis, dry eye syndrome and acne rosacea

409
Q

osmotic diuretic that may be used in patients with raised ICP

A

IV mannitol

410
Q

otitis media - Indications for antibiotics in children are:

A

Indications for antibiotics in children are:

Perforated eardrum
<2 years old and bilateral
Present for ≥4 days
<3 months old

411
Q

Examples of acyanotic heart disease

A

Ventricular septal defect
Coarctation of the aorta
Patent ductus arteriosus

412
Q

Types of brain tumours in children include:

A

Craniopharyngioma: originates from the pituitary gland and presents with homonymous hemianopia from pressure on the optic chiasm.

Astrocytoma: graded from 1 (pilocytic astrocytoma) to 4 (glioblastoma multiforme). Grade 1 tumours have an excellent prognosis (95% 5 year survival). Grade 4 tumours have a poor prognosis (20% 5 year survival). Grade 4 astrocytomas may cross the corpus callosum, giving a characteristic ‘butterfly glioma’ appearance on CT.

Ependymomas (tumours of the cells of the ventricular system)
Meningioma (tumours of arachnoid cells of the meninges)

413
Q

bronchiolitis obliterans organism

A

Whilst bronchiolitis obliterans is a relatively rare complication of bronchiolitis it is most commonly associated with adenovirus infection.

414
Q

complication of RSV

A

Hyponatraemia is commonly seen as a complication in bronchiolitis caused by respiratory syncytial virus (RSV).

415
Q

Congenital Adrenal Hyperplasia (CAH)

A

s a genetic defect in an enzyme in the synthesis of steroid hormones. Patients can present within a few weeks of birth, with evidence of vomiting and dehydration - suggests that this is the most common and also most severe of the CAH syndromes: classical “salt-wasting” 21-hydroxylase deficiency.

416
Q

Calculating paediatric doses using WETFLAG

A
Weight: (Age + 4) * 2
Energy: 4J/kg
Tube size: (Age/4) + 4
Fluids: 20ml/kg normal saline bolus (10ml/kg if septic, trauma or in heart failure)
Lorazepam: 0.1mg/kg
Adrenaline: 10 micrograms/kg
Glucose: 2ml/kg of 10% dextrose
417
Q

Glandular fever

A

EBV
illness worsened by antibiotics
Risk of speenic rupture -

418
Q

Meconium ileus is managed with

A

with ‘drip and suck’ (stomach drainage with an NG ryles tube, and IV fluids), along with enemas to remove the sticky meconium. In severe cases, surgery may be necessary.